+ All Categories
Home > Documents > Login ID:51A07053/Student Name:ANAND RAJ/Overall Score:0 · (i n sq. cm) of such a triangle?...

Login ID:51A07053/Student Name:ANAND RAJ/Overall Score:0 · (i n sq. cm) of such a triangle?...

Date post: 22-Mar-2020
Category:
Upload: others
View: 0 times
Download: 0 times
Share this document with a friend
78
Student ScoreCard Question: 1 Answer the question independently. Explanation: Question: 2 Answer the question independently. Quantitative Ability Score:0 Percentile:2 Data Interpretation & Logical Resoning Score:0 Percentile:4 Verbal Ability Score:0 Percentile:1 Overall: Overall Score:0 Percentile:6 1) 0 2) 2 3) 4 4) 6 1) N is less than 100 2) N is greater than 100 but less than 500 3) N is greater than 500 but less than 1000 Login ID:51A07053/Student Name:ANAND RAJ/Overall Score:0 © Copyright IMS Learning Resources Pvt.Ltd.,Mumbai.All copyrights to this material vestswith IMS Learning Resources Pvt.Ltd. No part of this materials either in part oras a whole shall be copied,printed,electronically reproduced,sold or distributed without the written consent of IMS Learing Resources Pvt.Ltd.and any such violation would entail initiation of suitable legal proceedings.
Transcript
Page 1: Login ID:51A07053/Student Name:ANAND RAJ/Overall Score:0 · (i n sq. cm) of such a triangle? Explanation: Question: 9 Answer the question independently. Eight colleagues, four men

Student ScoreCard

Question: 1Answer the question independently.

Explanation:

Question: 2Answer the question independently.

Quantitative Ability Score:0 Percentile:2

Data Interpretation & Logical Resoning Score:0 Percentile:4

Verbal Ability Score:0 Percentile:1

Overall: Overall Score:0 Percentile:6

1) 0

2) 2

3) 4

4) 6

1) N is less than 100

2) N is greater than 100 but less than 500

3) N is greater than 500 but less than 1000

Login ID:51A07053/Student Name:ANAND RAJ/Overall Score:0

© CopyrightIMS Learning Resources Pvt.Ltd.,Mumbai.All copyrights to this material vestswith IMS Learning Resources Pvt.Ltd.No part of this materials either in part oras a whole shall be copied,printed,electronically reproduced,sold or distributed without the writtenconsent of IMS Learing Resources Pvt.Ltd.and any such violation would entail initiation of suitable legal proceedings.

Page 2: Login ID:51A07053/Student Name:ANAND RAJ/Overall Score:0 · (i n sq. cm) of such a triangle? Explanation: Question: 9 Answer the question independently. Eight colleagues, four men

Explanation:

Question: 3Answer the question independently.

Explanation:

Question: 4Answer the question independently.

Barrack and Vladimir start running from the opposite ends of a linear track. Barrack's speed is two times the speed of Vladimir. If Barracktakes 4 seconds to reach the opposite end after meeting Vladimir, how much time does Vladimir take to reach the opposite end from theinstant he started running?

Explanation:

4) N is greater than 1000

1) 60,000 kg

2) 90,000 kg

3) 27,000 kg

4) 30,000 kg

1) 8 seconds

2) 12 seconds

3) 24 seconds

4) 16 seconds

Login ID:51A07053/Student Name:ANAND RAJ/Overall Score:0

© CopyrightIMS Learning Resources Pvt.Ltd.,Mumbai.All copyrights to this material vestswith IMS Learning Resources Pvt.Ltd.No part of this materials either in part oras a whole shall be copied,printed,electronically reproduced,sold or distributed without the writtenconsent of IMS Learing Resources Pvt.Ltd.and any such violation would entail initiation of suitable legal proceedings.

Page 3: Login ID:51A07053/Student Name:ANAND RAJ/Overall Score:0 · (i n sq. cm) of such a triangle? Explanation: Question: 9 Answer the question independently. Eight colleagues, four men

Question: 5Answer the question independently.A three-digit number in base 6 is represented as 'abc', where a, b and c are the three digits in the number. How many such numbers exist

such that the same number, when expressed in base 4, is '1abc'?

Explanation:

Question: 6Answer the question independently.

Explanation:

1) 0

2) 2

3) 4

4) 6

1) 4

2) 2

3) 5

4) Infinite

Login ID:51A07053/Student Name:ANAND RAJ/Overall Score:0

© CopyrightIMS Learning Resources Pvt.Ltd.,Mumbai.All copyrights to this material vestswith IMS Learning Resources Pvt.Ltd.No part of this materials either in part oras a whole shall be copied,printed,electronically reproduced,sold or distributed without the writtenconsent of IMS Learing Resources Pvt.Ltd.and any such violation would entail initiation of suitable legal proceedings.

Page 4: Login ID:51A07053/Student Name:ANAND RAJ/Overall Score:0 · (i n sq. cm) of such a triangle? Explanation: Question: 9 Answer the question independently. Eight colleagues, four men

Question: 7Answer the question independently.

Two friends, Mohan and Rohan, start a business by investing Rs. 150,000 and Rs. 200,000 respectively. After 6 months, Rohan withdrewsome amount of his investment. At the same time, a third friend, Gaurav, joined the business by investing the same amount which Rohanhad withdrawn. At the end of the year, Rohan and Gaurav distributed their profits in the ratio 5 : 3. If the total profit for the year was Rs.1.54 lakhs, what was the share of Mohan in the profit (in Rs)?

Explanation:

Question: 8Answer the question independently.The perimeter of a triangle, having length (in cm) of each side as integers, is 24. What can be said about the minimum value of the area

1) 66,000

2) 84,000

3) 96,000

4) 72,000

Login ID:51A07053/Student Name:ANAND RAJ/Overall Score:0

© CopyrightIMS Learning Resources Pvt.Ltd.,Mumbai.All copyrights to this material vestswith IMS Learning Resources Pvt.Ltd.No part of this materials either in part oras a whole shall be copied,printed,electronically reproduced,sold or distributed without the writtenconsent of IMS Learing Resources Pvt.Ltd.and any such violation would entail initiation of suitable legal proceedings.

Page 5: Login ID:51A07053/Student Name:ANAND RAJ/Overall Score:0 · (i n sq. cm) of such a triangle? Explanation: Question: 9 Answer the question independently. Eight colleagues, four men

(in sq. cm) of such a triangle?

Explanation:

Question: 9Answer the question independently.Eight colleagues, four men and four women, have booked a circular table for dining to celebrate the engagement of a woman in the

group, named Ramya. If Ramya prefers to be flanked on either side by only her female friends, how many seating arrangements arepossible?

Explanation:

1) It is less than 12

2) It is between 12 and 16

3) It is between 16 and 20

4) It is between 20 and 24

1) 720

2) 1080

3) 1540

4) 1440

Login ID:51A07053/Student Name:ANAND RAJ/Overall Score:0

© CopyrightIMS Learning Resources Pvt.Ltd.,Mumbai.All copyrights to this material vestswith IMS Learning Resources Pvt.Ltd.No part of this materials either in part oras a whole shall be copied,printed,electronically reproduced,sold or distributed without the writtenconsent of IMS Learing Resources Pvt.Ltd.and any such violation would entail initiation of suitable legal proceedings.

Page 6: Login ID:51A07053/Student Name:ANAND RAJ/Overall Score:0 · (i n sq. cm) of such a triangle? Explanation: Question: 9 Answer the question independently. Eight colleagues, four men

Question: 10Answer the question independently.

Explanation:

Question: 11Answer the question independently.

The total income through the tuition fees of a management school has two components – the first component is fixed and the secondcomponent varies directly with the number of students. The average tuition fee per student is Rs. 5 Lakhs when there are 30 students andRs. 6 Lakhs when there are 20 students. If the average tuition fee per student is Rs. 4 Lakhs, how many students are there?

Explanation:

1) 56

2) 60

3) 58

4) 62

1) 40

2) 45

3) 50

4) 60

Login ID:51A07053/Student Name:ANAND RAJ/Overall Score:0

© CopyrightIMS Learning Resources Pvt.Ltd.,Mumbai.All copyrights to this material vestswith IMS Learning Resources Pvt.Ltd.No part of this materials either in part oras a whole shall be copied,printed,electronically reproduced,sold or distributed without the writtenconsent of IMS Learing Resources Pvt.Ltd.and any such violation would entail initiation of suitable legal proceedings.

Page 7: Login ID:51A07053/Student Name:ANAND RAJ/Overall Score:0 · (i n sq. cm) of such a triangle? Explanation: Question: 9 Answer the question independently. Eight colleagues, four men

Question: 12Answer the question independently.

If the sum of the first 10 terms of a Geometric Progression is equal to the sum of the first 12 terms of the same progression and the 3 rd

term of the progression is 8, what is the 50thterm of the progression if it is known that no term in the progression is zero?

Explanation:

Question: 13Answer the question independently.How many three-digit odd numbers ending with 1 or 3 are possible such that the sum of the number and the number obtained by

reversing the digits is a four-digit number?

Explanation:

1) 4

2) 8

3) –8

4) More information is needed to answer the question

1) 40

2) 45

3) 50

4) 60

Login ID:51A07053/Student Name:ANAND RAJ/Overall Score:0

© CopyrightIMS Learning Resources Pvt.Ltd.,Mumbai.All copyrights to this material vestswith IMS Learning Resources Pvt.Ltd.No part of this materials either in part oras a whole shall be copied,printed,electronically reproduced,sold or distributed without the writtenconsent of IMS Learing Resources Pvt.Ltd.and any such violation would entail initiation of suitable legal proceedings.

Page 8: Login ID:51A07053/Student Name:ANAND RAJ/Overall Score:0 · (i n sq. cm) of such a triangle? Explanation: Question: 9 Answer the question independently. Eight colleagues, four men

Question: 14Answer the question independently.

Explanation:

Question: 15Answer the question independently.

How many whole number solutions exist to the equation a + b + c = 15 such that a 8, b 8 and c 8?

Explanation:

1) 8

2) 0

3) 4

4) 6

1) 136

2) 80

3) 108

4) 52

Login ID:51A07053/Student Name:ANAND RAJ/Overall Score:0

© CopyrightIMS Learning Resources Pvt.Ltd.,Mumbai.All copyrights to this material vestswith IMS Learning Resources Pvt.Ltd.No part of this materials either in part oras a whole shall be copied,printed,electronically reproduced,sold or distributed without the writtenconsent of IMS Learing Resources Pvt.Ltd.and any such violation would entail initiation of suitable legal proceedings.

Page 9: Login ID:51A07053/Student Name:ANAND RAJ/Overall Score:0 · (i n sq. cm) of such a triangle? Explanation: Question: 9 Answer the question independently. Eight colleagues, four men

Question: 16Answer the question independently.

There is an escalator going up from the ground floor to the first floor. Raghu is climbing up the escalator while Rajan is climbing down(from the first floor to the ground floor) on the same escalator. On a stationary escalator, the time taken by Rajan to climb 8 steps is sameas the time taken by Raghu to climb 6 steps. On the moving escalator, if Raghu takes 18 steps to reach the first floor while Rajan takes 60steps to reach the ground floor, how many steps are there on the escalator?

Explanation:

1) 24

2) 30

3) 36

4) 48

Login ID:51A07053/Student Name:ANAND RAJ/Overall Score:0

© CopyrightIMS Learning Resources Pvt.Ltd.,Mumbai.All copyrights to this material vestswith IMS Learning Resources Pvt.Ltd.No part of this materials either in part oras a whole shall be copied,printed,electronically reproduced,sold or distributed without the writtenconsent of IMS Learing Resources Pvt.Ltd.and any such violation would entail initiation of suitable legal proceedings.

Page 10: Login ID:51A07053/Student Name:ANAND RAJ/Overall Score:0 · (i n sq. cm) of such a triangle? Explanation: Question: 9 Answer the question independently. Eight colleagues, four men

HHence, [2].

Question: 17Answer the question independently.

Explanation:

1)

2)

3) 4

4)

Login ID:51A07053/Student Name:ANAND RAJ/Overall Score:0

© CopyrightIMS Learning Resources Pvt.Ltd.,Mumbai.All copyrights to this material vestswith IMS Learning Resources Pvt.Ltd.No part of this materials either in part oras a whole shall be copied,printed,electronically reproduced,sold or distributed without the writtenconsent of IMS Learing Resources Pvt.Ltd.and any such violation would entail initiation of suitable legal proceedings.

Page 11: Login ID:51A07053/Student Name:ANAND RAJ/Overall Score:0 · (i n sq. cm) of such a triangle? Explanation: Question: 9 Answer the question independently. Eight colleagues, four men

Question: 18Answer the question independently.What is the total number of even factors of 5280?

Explanation:

Question: 19Answer the question independently.

Suppose series a is 4, 11, 18 ... and series b is 2, 7, 12 .... Both series a and b contain 100 terms each. How many common elements doseries a and b have?

1) 8

2) 48

3) 40

4) 32

Login ID:51A07053/Student Name:ANAND RAJ/Overall Score:0

© CopyrightIMS Learning Resources Pvt.Ltd.,Mumbai.All copyrights to this material vestswith IMS Learning Resources Pvt.Ltd.No part of this materials either in part oras a whole shall be copied,printed,electronically reproduced,sold or distributed without the writtenconsent of IMS Learing Resources Pvt.Ltd.and any such violation would entail initiation of suitable legal proceedings.

Page 12: Login ID:51A07053/Student Name:ANAND RAJ/Overall Score:0 · (i n sq. cm) of such a triangle? Explanation: Question: 9 Answer the question independently. Eight colleagues, four men

Enter your response (as an integer) using the virtual keyboard.

[quizky-text]

Explanation:

Question: 20Answer the question independently.

How many points having both x and y coordinates as positive integers lie strictly within the region enclosed by the line 2x + 3y = 12 andthe two co-ordinate axes?

Enter your response (as an integer) using the virtual keyboard.

[quizky-text]

Explanation:

1) 14

2) 18

3) 19

4) 22

1) 7

2) 2

3) 4

4) 1

Login ID:51A07053/Student Name:ANAND RAJ/Overall Score:0

© CopyrightIMS Learning Resources Pvt.Ltd.,Mumbai.All copyrights to this material vestswith IMS Learning Resources Pvt.Ltd.No part of this materials either in part oras a whole shall be copied,printed,electronically reproduced,sold or distributed without the writtenconsent of IMS Learing Resources Pvt.Ltd.and any such violation would entail initiation of suitable legal proceedings.

Page 13: Login ID:51A07053/Student Name:ANAND RAJ/Overall Score:0 · (i n sq. cm) of such a triangle? Explanation: Question: 9 Answer the question independently. Eight colleagues, four men

Question: 21Answer the question independently.

Explanation:

1) 1.45

2) 1.35

3) 1.15

4) 1.25

Login ID:51A07053/Student Name:ANAND RAJ/Overall Score:0

© CopyrightIMS Learning Resources Pvt.Ltd.,Mumbai.All copyrights to this material vestswith IMS Learning Resources Pvt.Ltd.No part of this materials either in part oras a whole shall be copied,printed,electronically reproduced,sold or distributed without the writtenconsent of IMS Learing Resources Pvt.Ltd.and any such violation would entail initiation of suitable legal proceedings.

Page 14: Login ID:51A07053/Student Name:ANAND RAJ/Overall Score:0 · (i n sq. cm) of such a triangle? Explanation: Question: 9 Answer the question independently. Eight colleagues, four men

Question: 22Answer the question independently.

An unscrupulous milk seller bought certain quantity of pure milk. He first sold 20 litres of pure milk. He added 20 litres of water to theremaining quantity of milk. He then sold 20 litres of the milk-water mixture and added 20 litres of water to the mixture. He again sold 20litres of this milk-water mixture and added 20 litres of water to the mixture. Now, the ratio of milk to water is 8 : 19. How much quantity ofpure milk (in litres) did he buy in the first place?

Enter your response (as an integer) using the virtual keyboard.

[quizky-text]1) 60

2) 40

3) 20

Login ID:51A07053/Student Name:ANAND RAJ/Overall Score:0

© CopyrightIMS Learning Resources Pvt.Ltd.,Mumbai.All copyrights to this material vestswith IMS Learning Resources Pvt.Ltd.No part of this materials either in part oras a whole shall be copied,printed,electronically reproduced,sold or distributed without the writtenconsent of IMS Learing Resources Pvt.Ltd.and any such violation would entail initiation of suitable legal proceedings.

Page 15: Login ID:51A07053/Student Name:ANAND RAJ/Overall Score:0 · (i n sq. cm) of such a triangle? Explanation: Question: 9 Answer the question independently. Eight colleagues, four men

Explanation:

Question: 23Answer the question independently.

Gaurav, a resident of Jhumri Telaiya, is an avid reader and a member of Jhumri Telaiya Public Library. In the library, books on foursubjects are available, namely Sociology, History, Economics and Political Science. Every time he visits the library, he is equally likely topick a book on any of the four subjects. He visited the library four times in September. What is the probability that he picked one book oneach of the four subjects in September?

Explanation:

Question: 24Answer the question independently.

4) 70

1)

2)

3)

4)

Login ID:51A07053/Student Name:ANAND RAJ/Overall Score:0

© CopyrightIMS Learning Resources Pvt.Ltd.,Mumbai.All copyrights to this material vestswith IMS Learning Resources Pvt.Ltd.No part of this materials either in part oras a whole shall be copied,printed,electronically reproduced,sold or distributed without the writtenconsent of IMS Learing Resources Pvt.Ltd.and any such violation would entail initiation of suitable legal proceedings.

Page 16: Login ID:51A07053/Student Name:ANAND RAJ/Overall Score:0 · (i n sq. cm) of such a triangle? Explanation: Question: 9 Answer the question independently. Eight colleagues, four men

Explanation:

Question: 25Answer the question independently.and are the roots of the quadratic equation x2 + bx + c = 0 whereas and are the roots of the quadratic equation x2 + bx + (c + 3) = 0

such that the absolute value of the difference between and and the absolute value of the difference between and are 1. What is theabsolute value of the difference between and ?

Explanation:

1) 105°

2) 110°

3) 120°

4) 100°

1) 0

2) 4

3) 5

4) Cannot be determined

Login ID:51A07053/Student Name:ANAND RAJ/Overall Score:0

© CopyrightIMS Learning Resources Pvt.Ltd.,Mumbai.All copyrights to this material vestswith IMS Learning Resources Pvt.Ltd.No part of this materials either in part oras a whole shall be copied,printed,electronically reproduced,sold or distributed without the writtenconsent of IMS Learing Resources Pvt.Ltd.and any such violation would entail initiation of suitable legal proceedings.

Page 17: Login ID:51A07053/Student Name:ANAND RAJ/Overall Score:0 · (i n sq. cm) of such a triangle? Explanation: Question: 9 Answer the question independently. Eight colleagues, four men

Question: 26Answer the question independently.

How many four-digit even numbers are possible such that the difference between the digits in the thousands' and the hundreds' places is1?

Explanation:

Question: 27Answer the question independently.

Enter your response (as an integer) using the virtual keyboard.[quizky-text]

1) 850

2) 765

3) 935

4) 950

1) 3

2) 2

Login ID:51A07053/Student Name:ANAND RAJ/Overall Score:0

© CopyrightIMS Learning Resources Pvt.Ltd.,Mumbai.All copyrights to this material vestswith IMS Learning Resources Pvt.Ltd.No part of this materials either in part oras a whole shall be copied,printed,electronically reproduced,sold or distributed without the writtenconsent of IMS Learing Resources Pvt.Ltd.and any such violation would entail initiation of suitable legal proceedings.

Page 18: Login ID:51A07053/Student Name:ANAND RAJ/Overall Score:0 · (i n sq. cm) of such a triangle? Explanation: Question: 9 Answer the question independently. Eight colleagues, four men

Explanation:

Question: 28Answer the question independently.Maya can complete a piece of work in 53 days if she works alone. The work is started by Maya and on the second day, one friend, who is

two times as efficient as Maya, joins her to finish the work. On the third day, another friend, who is three times as efficient as Maya, joinsher to finish the work. Similarly on the fourth day, a friend, who is four times as efficient as Maya, joins her to finish the work. This processcontinues till the work is completed. On any given day, Maya and only one of her friends work. What is the minimum number of days inwhich the work will be completed?

Explanation:

3) 6

4) 1

1) 8

2) 9

3) 10

4) 11

Login ID:51A07053/Student Name:ANAND RAJ/Overall Score:0

© CopyrightIMS Learning Resources Pvt.Ltd.,Mumbai.All copyrights to this material vestswith IMS Learning Resources Pvt.Ltd.No part of this materials either in part oras a whole shall be copied,printed,electronically reproduced,sold or distributed without the writtenconsent of IMS Learing Resources Pvt.Ltd.and any such violation would entail initiation of suitable legal proceedings.

Page 19: Login ID:51A07053/Student Name:ANAND RAJ/Overall Score:0 · (i n sq. cm) of such a triangle? Explanation: Question: 9 Answer the question independently. Eight colleagues, four men

Question: 29Answer the question independently.

There are 20 regular polygons such that the first is a square, the second is a pentagon, the third is a hexagon, and so on. What is the sumof the total number of the diagonals of these 20 polygons?

Enter your response (as an integer) using the virtual keyboard.[quizky-text]

Explanation:

1) 1750

2) 1650

3) 1550

4) 1450

Login ID:51A07053/Student Name:ANAND RAJ/Overall Score:0

© CopyrightIMS Learning Resources Pvt.Ltd.,Mumbai.All copyrights to this material vestswith IMS Learning Resources Pvt.Ltd.No part of this materials either in part oras a whole shall be copied,printed,electronically reproduced,sold or distributed without the writtenconsent of IMS Learing Resources Pvt.Ltd.and any such violation would entail initiation of suitable legal proceedings.

Page 20: Login ID:51A07053/Student Name:ANAND RAJ/Overall Score:0 · (i n sq. cm) of such a triangle? Explanation: Question: 9 Answer the question independently. Eight colleagues, four men

Question: 30Answer the question independently.

If a natural number N has 15 factors, 2N has 18 factors and 3N has 20 factors, how many factors would the number 6N have?

Explanation:If a number has 15 factors, it will be of the form a2 b .4

If a = 2, N will be 22 b4 or 2N will be 23 b4 and the number of factors of 2N will be 20.If b = 2, N will be a2 24 or 2N will be 25 a2 and the number of factors of 2N will be 18.If neither a nor b is 2, 2N will be 21 a2 b4 and the number of factors of 2N will be 30.Therefore, the number is of the form 24 b .2

If b = 3, the number will be 24 32 or 3N = 24 33 and the number of factors of 3N will be 20.If b 3, the number 3N will be 31 24 b2 and the number of factors will be 30.Therefore, the number is of the form 24 32 and 6N is 25 33 and the number of factors = 24.Hence [2].

Question: 31Answer the question independently.

Explanation:

1) 22

2) 24

3) 36

4) Cannot be determined

1) 0

2) 1

3) 2

4) 3

Login ID:51A07053/Student Name:ANAND RAJ/Overall Score:0

© CopyrightIMS Learning Resources Pvt.Ltd.,Mumbai.All copyrights to this material vestswith IMS Learning Resources Pvt.Ltd.No part of this materials either in part oras a whole shall be copied,printed,electronically reproduced,sold or distributed without the writtenconsent of IMS Learing Resources Pvt.Ltd.and any such violation would entail initiation of suitable legal proceedings.

Page 21: Login ID:51A07053/Student Name:ANAND RAJ/Overall Score:0 · (i n sq. cm) of such a triangle? Explanation: Question: 9 Answer the question independently. Eight colleagues, four men

Question: 32Answer the question independently.

Explanation:

1) 2.4

2) 4.8

3) 3.6

4) 6

Login ID:51A07053/Student Name:ANAND RAJ/Overall Score:0

© CopyrightIMS Learning Resources Pvt.Ltd.,Mumbai.All copyrights to this material vestswith IMS Learning Resources Pvt.Ltd.No part of this materials either in part oras a whole shall be copied,printed,electronically reproduced,sold or distributed without the writtenconsent of IMS Learing Resources Pvt.Ltd.and any such violation would entail initiation of suitable legal proceedings.

Page 22: Login ID:51A07053/Student Name:ANAND RAJ/Overall Score:0 · (i n sq. cm) of such a triangle? Explanation: Question: 9 Answer the question independently. Eight colleagues, four men

Question: 33Answer the question independently.

A shopkeeper bought two pairs of jeans for Rs. 900 each. He sold the first pair at 8% profit and the second pair at 10% loss. The discounton the marked price of each pair was 40% and 70% respectively. By what percentage was the marked price of the second pair greaterthan that of the first?

Explanation:

1) 33.33%

2) 40%

3) 62.5%

4) 66.66%

Login ID:51A07053/Student Name:ANAND RAJ/Overall Score:0

© CopyrightIMS Learning Resources Pvt.Ltd.,Mumbai.All copyrights to this material vestswith IMS Learning Resources Pvt.Ltd.No part of this materials either in part oras a whole shall be copied,printed,electronically reproduced,sold or distributed without the writtenconsent of IMS Learing Resources Pvt.Ltd.and any such violation would entail initiation of suitable legal proceedings.

Page 23: Login ID:51A07053/Student Name:ANAND RAJ/Overall Score:0 · (i n sq. cm) of such a triangle? Explanation: Question: 9 Answer the question independently. Eight colleagues, four men

Question: 34Answer the question independently.

Enter your response (as an integer) using the virtual keyboard.[quizky-text]

Explanation:

Question: 35Refer to the data below and answer the questions that follow.

1) 2

2) 3

3) 4

4) 1

Login ID:51A07053/Student Name:ANAND RAJ/Overall Score:0

© CopyrightIMS Learning Resources Pvt.Ltd.,Mumbai.All copyrights to this material vestswith IMS Learning Resources Pvt.Ltd.No part of this materials either in part oras a whole shall be copied,printed,electronically reproduced,sold or distributed without the writtenconsent of IMS Learing Resources Pvt.Ltd.and any such violation would entail initiation of suitable legal proceedings.

Page 24: Login ID:51A07053/Student Name:ANAND RAJ/Overall Score:0 · (i n sq. cm) of such a triangle? Explanation: Question: 9 Answer the question independently. Eight colleagues, four men

In 2013, Chunilal, a wholesale trader in Mandsaur district, sold 10% of the wheat produced in the district in the wholesale market. On theother hand, in the same year, Manilal, another wholesale trader in Mandsaur district, sold 5% of the rice produced in the district in thewholesale market. Which of the following was the closest value of the ratio of the revenue earned by Chunilal and the revenue earned byManilal?

Explanation:

1) 2 : 3

2) 5 : 6

3) 1 : 1

4) 3 : 2

Login ID:51A07053/Student Name:ANAND RAJ/Overall Score:0

© CopyrightIMS Learning Resources Pvt.Ltd.,Mumbai.All copyrights to this material vestswith IMS Learning Resources Pvt.Ltd.No part of this materials either in part oras a whole shall be copied,printed,electronically reproduced,sold or distributed without the writtenconsent of IMS Learing Resources Pvt.Ltd.and any such violation would entail initiation of suitable legal proceedings.

Page 25: Login ID:51A07053/Student Name:ANAND RAJ/Overall Score:0 · (i n sq. cm) of such a triangle? Explanation: Question: 9 Answer the question independently. Eight colleagues, four men

Question: 36Refer to the data below and answer the questions that follow.

In 2013, Chunilal, the wholesale trader, sold wheat in the wholesale market to retail merchants and earned 20% profit margin. Retailmerchants in turn sold the wheat to individuals and earned 30% profit margin. What was the price paid by the individuals per kg of wheat?(Use data from the previous question, if required)

Explanation:

1) Rs. 20/kg

2) Rs. 21.84/kg

3) Rs. 18.2/kg

4) Cannot be determined

Login ID:51A07053/Student Name:ANAND RAJ/Overall Score:0

© CopyrightIMS Learning Resources Pvt.Ltd.,Mumbai.All copyrights to this material vestswith IMS Learning Resources Pvt.Ltd.No part of this materials either in part oras a whole shall be copied,printed,electronically reproduced,sold or distributed without the writtenconsent of IMS Learing Resources Pvt.Ltd.and any such violation would entail initiation of suitable legal proceedings.

Page 26: Login ID:51A07053/Student Name:ANAND RAJ/Overall Score:0 · (i n sq. cm) of such a triangle? Explanation: Question: 9 Answer the question independently. Eight colleagues, four men

Question: 37Refer to the data below and answer the questions that follow.

In 2015, due to drought, production of all the cereals dropped by 20% over their production in 2014 and the prices of all the cereals in thewholesale market increased by 20% over their prices in 2014. What was the revenue obtained by selling the cereals in the wholesalemarket in 2015 (in Rs. crores)?

Explanation:

Question: 38Refer to the data below and answer the questions that follow.

In 2011, the selling price per kg of rice in the wholesale market was 3 times that of corn in the wholesale market. Find the revenuegenerated by selling the entire quantity of rice produced in that year.

1) 460

2) 420

3) 500

4) 480

Login ID:51A07053/Student Name:ANAND RAJ/Overall Score:0

© CopyrightIMS Learning Resources Pvt.Ltd.,Mumbai.All copyrights to this material vestswith IMS Learning Resources Pvt.Ltd.No part of this materials either in part oras a whole shall be copied,printed,electronically reproduced,sold or distributed without the writtenconsent of IMS Learing Resources Pvt.Ltd.and any such violation would entail initiation of suitable legal proceedings.

Page 27: Login ID:51A07053/Student Name:ANAND RAJ/Overall Score:0 · (i n sq. cm) of such a triangle? Explanation: Question: 9 Answer the question independently. Eight colleagues, four men

Explanation:The price per kg rice in the year 2011 was Rs. 30 and the rice production for that year was 90000 × 1.06 = 95400 MT or 95400 thousandkg.Therefore, the revenue generated = 95400 × 30 thousand = Rs. 2,862,000 thousand. Hence [4].

Question: 39Refer to the data below and answer the questions that follow.

1) Rs. 1,956,000 Thousand

2) Rs. 3,542,000 Thousand

3) Rs. 5,389,000 Thousand

4) Rs. 2,862,000 Thousand

Login ID:51A07053/Student Name:ANAND RAJ/Overall Score:0

© CopyrightIMS Learning Resources Pvt.Ltd.,Mumbai.All copyrights to this material vestswith IMS Learning Resources Pvt.Ltd.No part of this materials either in part oras a whole shall be copied,printed,electronically reproduced,sold or distributed without the writtenconsent of IMS Learing Resources Pvt.Ltd.and any such violation would entail initiation of suitable legal proceedings.

Page 28: Login ID:51A07053/Student Name:ANAND RAJ/Overall Score:0 · (i n sq. cm) of such a triangle? Explanation: Question: 9 Answer the question independently. Eight colleagues, four men

Which colour is to the immediate right of the Blue part in any wheel?

Explanation:

1) Maroon

2) Pink

3) White

4) Red

Login ID:51A07053/Student Name:ANAND RAJ/Overall Score:0

© CopyrightIMS Learning Resources Pvt.Ltd.,Mumbai.All copyrights to this material vestswith IMS Learning Resources Pvt.Ltd.No part of this materials either in part oras a whole shall be copied,printed,electronically reproduced,sold or distributed without the writtenconsent of IMS Learing Resources Pvt.Ltd.and any such violation would entail initiation of suitable legal proceedings.

Page 29: Login ID:51A07053/Student Name:ANAND RAJ/Overall Score:0 · (i n sq. cm) of such a triangle? Explanation: Question: 9 Answer the question independently. Eight colleagues, four men

Login ID:51A07053/Student Name:ANAND RAJ/Overall Score:0

© CopyrightIMS Learning Resources Pvt.Ltd.,Mumbai.All copyrights to this material vestswith IMS Learning Resources Pvt.Ltd.No part of this materials either in part oras a whole shall be copied,printed,electronically reproduced,sold or distributed without the writtenconsent of IMS Learing Resources Pvt.Ltd.and any such violation would entail initiation of suitable legal proceedings.

Page 30: Login ID:51A07053/Student Name:ANAND RAJ/Overall Score:0 · (i n sq. cm) of such a triangle? Explanation: Question: 9 Answer the question independently. Eight colleagues, four men

Hence, [4].

Question: 40Refer to the data below and answer the questions that follow.

Which colour is diametrically opposite to the White part in any wheel?Enter your response (only the initial letter of the name of the colour) using the virtual keyboard.[quizky-text]

Explanation:

1) Y

2) G

3) O

4) B

Login ID:51A07053/Student Name:ANAND RAJ/Overall Score:0

© CopyrightIMS Learning Resources Pvt.Ltd.,Mumbai.All copyrights to this material vestswith IMS Learning Resources Pvt.Ltd.No part of this materials either in part oras a whole shall be copied,printed,electronically reproduced,sold or distributed without the writtenconsent of IMS Learing Resources Pvt.Ltd.and any such violation would entail initiation of suitable legal proceedings.

Page 31: Login ID:51A07053/Student Name:ANAND RAJ/Overall Score:0 · (i n sq. cm) of such a triangle? Explanation: Question: 9 Answer the question independently. Eight colleagues, four men

Login ID:51A07053/Student Name:ANAND RAJ/Overall Score:0

© CopyrightIMS Learning Resources Pvt.Ltd.,Mumbai.All copyrights to this material vestswith IMS Learning Resources Pvt.Ltd.No part of this materials either in part oras a whole shall be copied,printed,electronically reproduced,sold or distributed without the writtenconsent of IMS Learing Resources Pvt.Ltd.and any such violation would entail initiation of suitable legal proceedings.

Page 32: Login ID:51A07053/Student Name:ANAND RAJ/Overall Score:0 · (i n sq. cm) of such a triangle? Explanation: Question: 9 Answer the question independently. Eight colleagues, four men

Yellow is diametrically opposite to White. Therefore, the required answer is Y.

Question: 41Refer to the data below and answer the questions that follow.Which of the following combination of colours were at the same position?

Explanation:

1) Yellow-Green-White

2) Maroon-Yellow-Red

3) Pink-Maroon-Blue

4) Blue-Green-Orange

Login ID:51A07053/Student Name:ANAND RAJ/Overall Score:0

© CopyrightIMS Learning Resources Pvt.Ltd.,Mumbai.All copyrights to this material vestswith IMS Learning Resources Pvt.Ltd.No part of this materials either in part oras a whole shall be copied,printed,electronically reproduced,sold or distributed without the writtenconsent of IMS Learing Resources Pvt.Ltd.and any such violation would entail initiation of suitable legal proceedings.

Page 33: Login ID:51A07053/Student Name:ANAND RAJ/Overall Score:0 · (i n sq. cm) of such a triangle? Explanation: Question: 9 Answer the question independently. Eight colleagues, four men

Login ID:51A07053/Student Name:ANAND RAJ/Overall Score:0

© CopyrightIMS Learning Resources Pvt.Ltd.,Mumbai.All copyrights to this material vestswith IMS Learning Resources Pvt.Ltd.No part of this materials either in part oras a whole shall be copied,printed,electronically reproduced,sold or distributed without the writtenconsent of IMS Learing Resources Pvt.Ltd.and any such violation would entail initiation of suitable legal proceedings.

Page 34: Login ID:51A07053/Student Name:ANAND RAJ/Overall Score:0 · (i n sq. cm) of such a triangle? Explanation: Question: 9 Answer the question independently. Eight colleagues, four men

Hence, [2].

Question: 42Refer to the data below and answer the questions that follow.Which of the following combination of colours were not at the same position?

Explanation:

1) Pink-Maroon-Blue

2) Green-Red-Maroon

3) White-Maroon-Green

4) Blue-White-Orange

Login ID:51A07053/Student Name:ANAND RAJ/Overall Score:0

© CopyrightIMS Learning Resources Pvt.Ltd.,Mumbai.All copyrights to this material vestswith IMS Learning Resources Pvt.Ltd.No part of this materials either in part oras a whole shall be copied,printed,electronically reproduced,sold or distributed without the writtenconsent of IMS Learing Resources Pvt.Ltd.and any such violation would entail initiation of suitable legal proceedings.

Page 35: Login ID:51A07053/Student Name:ANAND RAJ/Overall Score:0 · (i n sq. cm) of such a triangle? Explanation: Question: 9 Answer the question independently. Eight colleagues, four men

Login ID:51A07053/Student Name:ANAND RAJ/Overall Score:0

© CopyrightIMS Learning Resources Pvt.Ltd.,Mumbai.All copyrights to this material vestswith IMS Learning Resources Pvt.Ltd.No part of this materials either in part oras a whole shall be copied,printed,electronically reproduced,sold or distributed without the writtenconsent of IMS Learing Resources Pvt.Ltd.and any such violation would entail initiation of suitable legal proceedings.

Page 36: Login ID:51A07053/Student Name:ANAND RAJ/Overall Score:0 · (i n sq. cm) of such a triangle? Explanation: Question: 9 Answer the question independently. Eight colleagues, four men

Hence, [1].

Question: 43Refer to the data below and answer the questions that follow.

Machine for which of the following steps was idle (from 8.00 a.m. onwards) for the maximum time till it began processing the 4thbatch?

Login ID:51A07053/Student Name:ANAND RAJ/Overall Score:0

© CopyrightIMS Learning Resources Pvt.Ltd.,Mumbai.All copyrights to this material vestswith IMS Learning Resources Pvt.Ltd.No part of this materials either in part oras a whole shall be copied,printed,electronically reproduced,sold or distributed without the writtenconsent of IMS Learing Resources Pvt.Ltd.and any such violation would entail initiation of suitable legal proceedings.

Page 37: Login ID:51A07053/Student Name:ANAND RAJ/Overall Score:0 · (i n sq. cm) of such a triangle? Explanation: Question: 9 Answer the question independently. Eight colleagues, four men

Explanation:

Hence [4].

Question: 44Refer to the data below and answer the questions that follow.

How many boxes of biscuits were produced by 9.09 a.m. if 100 kg of total ingredients was used in Step 1 for each batch?Enter your response (as an integer) using the virtual keyboard.[quizky-text]

Explanation:

1) Step 1

2) Step 2

3) Step 3

4) Step 4

1) 1045

2) 1145

3) 1047

4) 1245

Login ID:51A07053/Student Name:ANAND RAJ/Overall Score:0

© CopyrightIMS Learning Resources Pvt.Ltd.,Mumbai.All copyrights to this material vestswith IMS Learning Resources Pvt.Ltd.No part of this materials either in part oras a whole shall be copied,printed,electronically reproduced,sold or distributed without the writtenconsent of IMS Learing Resources Pvt.Ltd.and any such violation would entail initiation of suitable legal proceedings.

Page 38: Login ID:51A07053/Student Name:ANAND RAJ/Overall Score:0 · (i n sq. cm) of such a triangle? Explanation: Question: 9 Answer the question independently. Eight colleagues, four men

Question: 45Refer to the data below and answer the questions that follow.

Due to the delay in the start of the fourth batch, the output of the fourth batch was delayed by:

Explanation:

1) 10 minutes

2) 1 minute

3) 5 minutes

4) None of these

Login ID:51A07053/Student Name:ANAND RAJ/Overall Score:0

© CopyrightIMS Learning Resources Pvt.Ltd.,Mumbai.All copyrights to this material vestswith IMS Learning Resources Pvt.Ltd.No part of this materials either in part oras a whole shall be copied,printed,electronically reproduced,sold or distributed without the writtenconsent of IMS Learing Resources Pvt.Ltd.and any such violation would entail initiation of suitable legal proceedings.

Page 39: Login ID:51A07053/Student Name:ANAND RAJ/Overall Score:0 · (i n sq. cm) of such a triangle? Explanation: Question: 9 Answer the question independently. Eight colleagues, four men

Hence [1].

Question: 46Refer to the data below and answer the questions that follow.

Which of the following was the scenario at 8.50 am?

Explanation:

Hence [1].

1)Machine for step 1 was idle, Machine for step 2 was processing 3rdbatch, Machine for step 3 was processing 2ndbatch and

Machine for step 4 was idle.

2)Machine for step 1 was processing 4thbatch, Machine for step 2 was processing 3rdbatch, Machine for step 3 was

processing 2ndbatch and Machine for step 4 was idle.

3)Machine for step 1 was idle, Machine for step 2 was processing 4thbatch, Machine for step 3 was processing 3rdbatch and

Machine for step 4 was idle.

4)Machine for step 1 was processing 3rdbatch, Machine for step 2 was processing 2nd batch, Machine for step 3 was idle

and Machine for step 4 was also idle.

Login ID:51A07053/Student Name:ANAND RAJ/Overall Score:0

© CopyrightIMS Learning Resources Pvt.Ltd.,Mumbai.All copyrights to this material vestswith IMS Learning Resources Pvt.Ltd.No part of this materials either in part oras a whole shall be copied,printed,electronically reproduced,sold or distributed without the writtenconsent of IMS Learing Resources Pvt.Ltd.and any such violation would entail initiation of suitable legal proceedings.

Page 40: Login ID:51A07053/Student Name:ANAND RAJ/Overall Score:0 · (i n sq. cm) of such a triangle? Explanation: Question: 9 Answer the question independently. Eight colleagues, four men

Question: 47Refer to the data below and answer the questions that follow.

Agent Bond belongs to ________.

Explanation:

1) Country I

2) Country C

3) Country A

4) Cannot be determined

Login ID:51A07053/Student Name:ANAND RAJ/Overall Score:0

© CopyrightIMS Learning Resources Pvt.Ltd.,Mumbai.All copyrights to this material vestswith IMS Learning Resources Pvt.Ltd.No part of this materials either in part oras a whole shall be copied,printed,electronically reproduced,sold or distributed without the writtenconsent of IMS Learing Resources Pvt.Ltd.and any such violation would entail initiation of suitable legal proceedings.

Page 41: Login ID:51A07053/Student Name:ANAND RAJ/Overall Score:0 · (i n sq. cm) of such a triangle? Explanation: Question: 9 Answer the question independently. Eight colleagues, four men

Hence [2]

Question: 48Refer to the data below and answer the questions that follow.

Agent Hulk is spying Country ________.

Login ID:51A07053/Student Name:ANAND RAJ/Overall Score:0

© CopyrightIMS Learning Resources Pvt.Ltd.,Mumbai.All copyrights to this material vestswith IMS Learning Resources Pvt.Ltd.No part of this materials either in part oras a whole shall be copied,printed,electronically reproduced,sold or distributed without the writtenconsent of IMS Learing Resources Pvt.Ltd.and any such violation would entail initiation of suitable legal proceedings.

Page 42: Login ID:51A07053/Student Name:ANAND RAJ/Overall Score:0 · (i n sq. cm) of such a triangle? Explanation: Question: 9 Answer the question independently. Eight colleagues, four men

Enter your response (as a letter) using the virtual keyboard.[quizky-text]

Explanation:

1) D

2) A

3) E

4) B

Login ID:51A07053/Student Name:ANAND RAJ/Overall Score:0

© CopyrightIMS Learning Resources Pvt.Ltd.,Mumbai.All copyrights to this material vestswith IMS Learning Resources Pvt.Ltd.No part of this materials either in part oras a whole shall be copied,printed,electronically reproduced,sold or distributed without the writtenconsent of IMS Learing Resources Pvt.Ltd.and any such violation would entail initiation of suitable legal proceedings.

Page 43: Login ID:51A07053/Student Name:ANAND RAJ/Overall Score:0 · (i n sq. cm) of such a triangle? Explanation: Question: 9 Answer the question independently. Eight colleagues, four men

The required answer is D.

Question: 49Refer to the data below and answer the questions that follow.

Agent Charlie is spying ________.

Explanation:

1) Country B

2) Country I

3) Country E

4) Cannot be determined

Login ID:51A07053/Student Name:ANAND RAJ/Overall Score:0

© CopyrightIMS Learning Resources Pvt.Ltd.,Mumbai.All copyrights to this material vestswith IMS Learning Resources Pvt.Ltd.No part of this materials either in part oras a whole shall be copied,printed,electronically reproduced,sold or distributed without the writtenconsent of IMS Learing Resources Pvt.Ltd.and any such violation would entail initiation of suitable legal proceedings.

Page 44: Login ID:51A07053/Student Name:ANAND RAJ/Overall Score:0 · (i n sq. cm) of such a triangle? Explanation: Question: 9 Answer the question independently. Eight colleagues, four men

Hence [4]

Question: 50Refer to the data below and answer the questions that follow.Agent ________ is from country A.

Login ID:51A07053/Student Name:ANAND RAJ/Overall Score:0

© CopyrightIMS Learning Resources Pvt.Ltd.,Mumbai.All copyrights to this material vestswith IMS Learning Resources Pvt.Ltd.No part of this materials either in part oras a whole shall be copied,printed,electronically reproduced,sold or distributed without the writtenconsent of IMS Learing Resources Pvt.Ltd.and any such violation would entail initiation of suitable legal proceedings.

Page 45: Login ID:51A07053/Student Name:ANAND RAJ/Overall Score:0 · (i n sq. cm) of such a triangle? Explanation: Question: 9 Answer the question independently. Eight colleagues, four men

Explanation:

1) Engage

2) Gordon

3) Bond

4) None of these

Login ID:51A07053/Student Name:ANAND RAJ/Overall Score:0

© CopyrightIMS Learning Resources Pvt.Ltd.,Mumbai.All copyrights to this material vestswith IMS Learning Resources Pvt.Ltd.No part of this materials either in part oras a whole shall be copied,printed,electronically reproduced,sold or distributed without the writtenconsent of IMS Learing Resources Pvt.Ltd.and any such violation would entail initiation of suitable legal proceedings.

Page 46: Login ID:51A07053/Student Name:ANAND RAJ/Overall Score:0 · (i n sq. cm) of such a triangle? Explanation: Question: 9 Answer the question independently. Eight colleagues, four men

Hence [1]

Question: 51Refer to the data below and answer the questions that follow.

How many students are there in class X of the school?Enter your response (as an integer) using the virtual keyboard.[quizky-text]

Explanation:

1) 116

2) 95

3) 105

4) 111

Login ID:51A07053/Student Name:ANAND RAJ/Overall Score:0

© CopyrightIMS Learning Resources Pvt.Ltd.,Mumbai.All copyrights to this material vestswith IMS Learning Resources Pvt.Ltd.No part of this materials either in part oras a whole shall be copied,printed,electronically reproduced,sold or distributed without the writtenconsent of IMS Learing Resources Pvt.Ltd.and any such violation would entail initiation of suitable legal proceedings.

Page 47: Login ID:51A07053/Student Name:ANAND RAJ/Overall Score:0 · (i n sq. cm) of such a triangle? Explanation: Question: 9 Answer the question independently. Eight colleagues, four men

The required answer is 13 + 18+5+15+2+4+3+4+6+11+9+11+11+4 = 116.

Question: 52Refer to the data below and answer the questions that follow.

How many girls like only Math and Chemistry?Enter your response (as an integer) using the virtual keyboard.[quizky-text]

Explanation:

1) 15

2) 19

3) 18

4) 11

Login ID:51A07053/Student Name:ANAND RAJ/Overall Score:0

© CopyrightIMS Learning Resources Pvt.Ltd.,Mumbai.All copyrights to this material vestswith IMS Learning Resources Pvt.Ltd.No part of this materials either in part oras a whole shall be copied,printed,electronically reproduced,sold or distributed without the writtenconsent of IMS Learing Resources Pvt.Ltd.and any such violation would entail initiation of suitable legal proceedings.

Page 48: Login ID:51A07053/Student Name:ANAND RAJ/Overall Score:0 · (i n sq. cm) of such a triangle? Explanation: Question: 9 Answer the question independently. Eight colleagues, four men

The required answer is 15.

Question: 53Refer to the data below and answer the questions that follow.

What is the ratio of the number of boys to the number of girls who like only one subject?

Explanation:

1) 9 : 10

2) 10 : 11

3) 11 : 12

4) 11 : 10

Login ID:51A07053/Student Name:ANAND RAJ/Overall Score:0

© CopyrightIMS Learning Resources Pvt.Ltd.,Mumbai.All copyrights to this material vestswith IMS Learning Resources Pvt.Ltd.No part of this materials either in part oras a whole shall be copied,printed,electronically reproduced,sold or distributed without the writtenconsent of IMS Learing Resources Pvt.Ltd.and any such violation would entail initiation of suitable legal proceedings.

Page 49: Login ID:51A07053/Student Name:ANAND RAJ/Overall Score:0 · (i n sq. cm) of such a triangle? Explanation: Question: 9 Answer the question independently. Eight colleagues, four men

Hence, [2].

Question: 54Refer to the data below and answer the questions that follow.

Each option consists of a pair of numbers. Which of these pairs has the two numbers not equal to each other?

Explanation:

1) Number of boys who like only Physics, Number of girls who like only Chemistry

2) Number of girls who like only Physics and Chemistry, Number of girls who like only Chemistry

3) Number of boys who like only Chemistry, Number of students who like only Math and Physics

4) Number of boys who like only Math and Chemistry, Number of girls who like only Physics

Login ID:51A07053/Student Name:ANAND RAJ/Overall Score:0

© CopyrightIMS Learning Resources Pvt.Ltd.,Mumbai.All copyrights to this material vestswith IMS Learning Resources Pvt.Ltd.No part of this materials either in part oras a whole shall be copied,printed,electronically reproduced,sold or distributed without the writtenconsent of IMS Learing Resources Pvt.Ltd.and any such violation would entail initiation of suitable legal proceedings.

Page 50: Login ID:51A07053/Student Name:ANAND RAJ/Overall Score:0 · (i n sq. cm) of such a triangle? Explanation: Question: 9 Answer the question independently. Eight colleagues, four men

Hence, [4].

Question: 55Refer to the data below and answer the questions that follow.

Login ID:51A07053/Student Name:ANAND RAJ/Overall Score:0

© CopyrightIMS Learning Resources Pvt.Ltd.,Mumbai.All copyrights to this material vestswith IMS Learning Resources Pvt.Ltd.No part of this materials either in part oras a whole shall be copied,printed,electronically reproduced,sold or distributed without the writtenconsent of IMS Learing Resources Pvt.Ltd.and any such violation would entail initiation of suitable legal proceedings.

Page 51: Login ID:51A07053/Student Name:ANAND RAJ/Overall Score:0 · (i n sq. cm) of such a triangle? Explanation: Question: 9 Answer the question independently. Eight colleagues, four men

Which model had the lowest price per handset in 2014?

Explanation:

1) Model B

2) Model C

3) Model E

4) Cannot be determined

Login ID:51A07053/Student Name:ANAND RAJ/Overall Score:0

© CopyrightIMS Learning Resources Pvt.Ltd.,Mumbai.All copyrights to this material vestswith IMS Learning Resources Pvt.Ltd.No part of this materials either in part oras a whole shall be copied,printed,electronically reproduced,sold or distributed without the writtenconsent of IMS Learing Resources Pvt.Ltd.and any such violation would entail initiation of suitable legal proceedings.

Page 52: Login ID:51A07053/Student Name:ANAND RAJ/Overall Score:0 · (i n sq. cm) of such a triangle? Explanation: Question: 9 Answer the question independently. Eight colleagues, four men

Question: 56Refer to the data below and answer the questions that follow.

What was the ratio of the price per handset of model A and that of model D?

Explanation:

Question: 57Refer to the data below and answer the questions that follow.

If the price per handset of model D in 2014 was Rs. 12,000, what was the price per handset of model B in 2014 (in Rs.)?Enter your response (rounded off to the nearest multiple of 10) using the virtual keyboard.[quizky-text]

1) 2 : 1

2) 5 : 2

3) 3 : 1

4) Cannot be determined

1) 14000

2) 30000

3) 40000

Login ID:51A07053/Student Name:ANAND RAJ/Overall Score:0

© CopyrightIMS Learning Resources Pvt.Ltd.,Mumbai.All copyrights to this material vestswith IMS Learning Resources Pvt.Ltd.No part of this materials either in part oras a whole shall be copied,printed,electronically reproduced,sold or distributed without the writtenconsent of IMS Learing Resources Pvt.Ltd.and any such violation would entail initiation of suitable legal proceedings.

Page 53: Login ID:51A07053/Student Name:ANAND RAJ/Overall Score:0 · (i n sq. cm) of such a triangle? Explanation: Question: 9 Answer the question independently. Eight colleagues, four men

Explanation:

Question: 58Refer to the data below and answer the questions that follow.

If it is known that the total revenue of the company in 2014 was Rs. 21 crores, how many handsets of model C were sold by the company?(Use data from the previous question)

Explanation:

Question: 59Refer to the data below and answer the questions that follow.At the eleventh hour, Pappu got his train ticket confirmed on Maharaja Express. He was surprised to see that the notification messagefrom IRTC (Indian Rail Transport Corporation) indicated only the train number and the seat number, but there was no mention of thecompartment number. On reaching the station, he saw that there were 9 compartments numbered S1 to S9 in that order and the enginewas attached to compartment S1. He checked exactly 5 compartments before he found his compartment. In each of these 5compartments, there was someone sitting in his seat number.The train being special, each compartment was painted with a differentcolour.

Pappu checked the Red compartment immediately before he checked S9. He checked S9 immediately before he checked S4, whereinhis seat number was taken by Charlie.S6 was the last compartment he checked, wherein his seat number was allotted to Emma.He checked the Yellow and Pink compartments as well, the Pink one being 4 compartments further away from the engine than theYellow one.Immediately after checking the Green compartment, Pappu ran 2 compartments towards the engine side and found his seat number

4) 20000

1) 4,500

2) 2,000

3) 5,000

4) Cannot be determined

Login ID:51A07053/Student Name:ANAND RAJ/Overall Score:0

© CopyrightIMS Learning Resources Pvt.Ltd.,Mumbai.All copyrights to this material vestswith IMS Learning Resources Pvt.Ltd.No part of this materials either in part oras a whole shall be copied,printed,electronically reproduced,sold or distributed without the writtenconsent of IMS Learing Resources Pvt.Ltd.and any such violation would entail initiation of suitable legal proceedings.

Page 54: Login ID:51A07053/Student Name:ANAND RAJ/Overall Score:0 · (i n sq. cm) of such a triangle? Explanation: Question: 9 Answer the question independently. Eight colleagues, four men

being occupied by Danny.Immediately after finding that his seat number in a compartment was occupied by Annie, Pappu ran 2 compartments further away fromthe engine to check the Blue compartment.Boyle was another person whom Pappu found to be occupying his seat number in one of the compartments.

Who was in the Pink compartment?

Explanation:

1) Emma

2) Annie

3) Danny

4) Charlie

Login ID:51A07053/Student Name:ANAND RAJ/Overall Score:0

© CopyrightIMS Learning Resources Pvt.Ltd.,Mumbai.All copyrights to this material vestswith IMS Learning Resources Pvt.Ltd.No part of this materials either in part oras a whole shall be copied,printed,electronically reproduced,sold or distributed without the writtenconsent of IMS Learing Resources Pvt.Ltd.and any such violation would entail initiation of suitable legal proceedings.

Page 55: Login ID:51A07053/Student Name:ANAND RAJ/Overall Score:0 · (i n sq. cm) of such a triangle? Explanation: Question: 9 Answer the question independently. Eight colleagues, four men

Hence [1]

Question: 60Refer to the data below and answer the questions that follow.Which of the following combinations is correct?

Explanation:

1) Annie, S7, Red

2) Danny, S4, Blue

3) Boyle, S9, Pink

4) Charlie, S6, Yellow

Login ID:51A07053/Student Name:ANAND RAJ/Overall Score:0

© CopyrightIMS Learning Resources Pvt.Ltd.,Mumbai.All copyrights to this material vestswith IMS Learning Resources Pvt.Ltd.No part of this materials either in part oras a whole shall be copied,printed,electronically reproduced,sold or distributed without the writtenconsent of IMS Learing Resources Pvt.Ltd.and any such violation would entail initiation of suitable legal proceedings.

Page 56: Login ID:51A07053/Student Name:ANAND RAJ/Overall Score:0 · (i n sq. cm) of such a triangle? Explanation: Question: 9 Answer the question independently. Eight colleagues, four men

Hence [1]

Login ID:51A07053/Student Name:ANAND RAJ/Overall Score:0

© CopyrightIMS Learning Resources Pvt.Ltd.,Mumbai.All copyrights to this material vestswith IMS Learning Resources Pvt.Ltd.No part of this materials either in part oras a whole shall be copied,printed,electronically reproduced,sold or distributed without the writtenconsent of IMS Learing Resources Pvt.Ltd.and any such violation would entail initiation of suitable legal proceedings.

Page 57: Login ID:51A07053/Student Name:ANAND RAJ/Overall Score:0 · (i n sq. cm) of such a triangle? Explanation: Question: 9 Answer the question independently. Eight colleagues, four men

Question: 61Refer to the data below and answer the questions that follow.Who did Pappu find first occupying his seat number?

Explanation:

1) Boyle

2) Charlie

3) Annie

4) None of these

Login ID:51A07053/Student Name:ANAND RAJ/Overall Score:0

© CopyrightIMS Learning Resources Pvt.Ltd.,Mumbai.All copyrights to this material vestswith IMS Learning Resources Pvt.Ltd.No part of this materials either in part oras a whole shall be copied,printed,electronically reproduced,sold or distributed without the writtenconsent of IMS Learing Resources Pvt.Ltd.and any such violation would entail initiation of suitable legal proceedings.

Page 58: Login ID:51A07053/Student Name:ANAND RAJ/Overall Score:0 · (i n sq. cm) of such a triangle? Explanation: Question: 9 Answer the question independently. Eight colleagues, four men

Hence [3]

Question: 62Refer to the data below and answer the questions that follow.Which among the following is the correct order of colours starting from the engine side?

Explanation:

1) Pink,Yellow, Green, Blue, Red

2) Yellow, Green, Pink, Red, Blue

3) Red, Blue, Green, Pink, Yellow

4) Red, Blue, Green, Yellow, Pink

Login ID:51A07053/Student Name:ANAND RAJ/Overall Score:0

© CopyrightIMS Learning Resources Pvt.Ltd.,Mumbai.All copyrights to this material vestswith IMS Learning Resources Pvt.Ltd.No part of this materials either in part oras a whole shall be copied,printed,electronically reproduced,sold or distributed without the writtenconsent of IMS Learing Resources Pvt.Ltd.and any such violation would entail initiation of suitable legal proceedings.

Page 59: Login ID:51A07053/Student Name:ANAND RAJ/Overall Score:0 · (i n sq. cm) of such a triangle? Explanation: Question: 9 Answer the question independently. Eight colleagues, four men

Hence [2]

Login ID:51A07053/Student Name:ANAND RAJ/Overall Score:0

© CopyrightIMS Learning Resources Pvt.Ltd.,Mumbai.All copyrights to this material vestswith IMS Learning Resources Pvt.Ltd.No part of this materials either in part oras a whole shall be copied,printed,electronically reproduced,sold or distributed without the writtenconsent of IMS Learing Resources Pvt.Ltd.and any such violation would entail initiation of suitable legal proceedings.

Page 60: Login ID:51A07053/Student Name:ANAND RAJ/Overall Score:0 · (i n sq. cm) of such a triangle? Explanation: Question: 9 Answer the question independently. Eight colleagues, four men

Question: 63Refer to the data below and answer the questions that follow.

If box F fits inside box D, which is the largest box?

Explanation:

Box D Box F and Box D Box A. We cannot determine which one out of boxes B and D is larger, but it is certain that one of them isthe largest. Hence, [4].

Question: 64Refer to the data below and answer the questions that follow.

Which of the following statements is definitely false? (Use the information from the previous question)

Explanation:

Since box B is larger than box G, it cannot fit inside box G. Hence, [2].

Question: 65Refer to the data below and answer the questions that follow.

If box C fits inside box G, which is the fifth largest box among the seven boxes? (Use the information from the previous question)

1) B

2) D

3) A

4) Either B or D

1) Box F fits inside box A.

2) Box B fits inside box G.

3) Box E fits inside box G.

4) Box A fits inside box F.

1) G

2) C

3) F

4) Cannot be determined

Login ID:51A07053/Student Name:ANAND RAJ/Overall Score:0

© CopyrightIMS Learning Resources Pvt.Ltd.,Mumbai.All copyrights to this material vestswith IMS Learning Resources Pvt.Ltd.No part of this materials either in part oras a whole shall be copied,printed,electronically reproduced,sold or distributed without the writtenconsent of IMS Learing Resources Pvt.Ltd.and any such violation would entail initiation of suitable legal proceedings.

Page 61: Login ID:51A07053/Student Name:ANAND RAJ/Overall Score:0 · (i n sq. cm) of such a triangle? Explanation: Question: 9 Answer the question independently. Eight colleagues, four men

Explanation:

Question: 66Refer to the data below and answer the questions that follow.

If the smallest box is put inside the second smallest box, the second smallest box is put inside the third smallest box . . . the secondlargest box is put inside the largest box, which among the following can have the highest value? (Use the information from the previousquestion)

Explanation:

The boxes in descending order are:[Box B/Box D] [Box D/Box B] [Box A/Box F] [Box F/Box A] [Box G] [Box C] [Box E]Option 1: The number of boxes outside F = 2 or 3Option 2: The number of boxes inside A = 3 or 4Option 3: The number of boxes inside D = 5 or 6Option 4: The number of boxes outside C = 5The maximum possible value is 6 for option 3.Hence, [3].

Question: 67In the following question, there are sentences that form a paragraph. Identify the sentence(s) or part(s) of sentence(s) that is/arecorrect in terms of grammar and usage (including spelling, punctuation and logical consistency). Enter in the box providedbelow the letters corresponding only to the correct sentences in alphabetical order.Note: Use the virtual keyboard provided to enter your answer.A. Google has announced a new typeface for Google Books: Literata.B. Replacing Droid Serif as the default typeface for e-books via Google Books, Literata is ‘perfect for long reads on all devices’, accordingto Google.C. Created in conjunction with type-design company TypeTogether, Google began work on Literata in April 2014.D. The goal was a ‘new book typeface would provide an outstanding reading experience on a whole range of devices and high resolutionscreens running different rendering technologies’.E.Literata will also ‘establish a recognizable visual identity’ for Google Books, distinguishing it from other popular e-reading mediums like

1) The number of boxes outside F.

2) The number of boxes inside A.

3) The number of boxes inside D.

4) The number of boxes outside C.

Login ID:51A07053/Student Name:ANAND RAJ/Overall Score:0

© CopyrightIMS Learning Resources Pvt.Ltd.,Mumbai.All copyrights to this material vestswith IMS Learning Resources Pvt.Ltd.No part of this materials either in part oras a whole shall be copied,printed,electronically reproduced,sold or distributed without the writtenconsent of IMS Learing Resources Pvt.Ltd.and any such violation would entail initiation of suitable legal proceedings.

Page 62: Login ID:51A07053/Student Name:ANAND RAJ/Overall Score:0 · (i n sq. cm) of such a triangle? Explanation: Question: 9 Answer the question independently. Eight colleagues, four men

Nook or Kindle.[quizky-text]

Explanation:There is a misplaced modifier in C: the modifier 'Created in conjunction with type-design company TypeTogether' refers to 'Literata', not'Google', so 'Literata' should be placed immediately adjacent to the modifier. This sentence should be rephrased as follows: 'In April 2014,Google began work on Literata, which was created in conjunction with type-design company TypeTogether.' The relative pronoun 'that' ismissing after 'typeface' in D. The other three sentences – A, B and E – are grammatically correct. Hence, ABE.

Question: 68In the following question, there are sentences that form a paragraph. Identify the sentence(s) or part(s) of sentence(s) that is/are

correct in terms of grammar and usage (including spelling, punctuation and logical consistency). Enter in the box providedbelow the letters corresponding only to the correct sentences in alphabetical order.Note: Use the virtual keyboard provided to enter your answer.A. Instead of archaeological layers have being removed with pickaxes (or even explosives!) as in the past, nowadays each layer iscarefully shovelled, scraped or brushed away,

B. and everything is seived, so as not to lose any scrap of information the earth might hold.

C. For instance, down the 'Pit of Bones' at Atapuerca, Spain, a chamber deep inside a cave,

D. that hold the skeletons of scores of people from at least 200,000 years ago, the excavators remove only about 10 inches of soil eachJuly.

E. This yields around 300 human bones, which are all they can handle since each has to be cleaned and hardened and conserved.

[quizky-text]

Explanation:There is a tense error in A: instead of 'have being removed', the correct form of the verb should be 'having been removed' or simply 'being

removed'. In B, the word 'sieved' is incorrectly spelt. There is a subject-verb disagreement in D: the subject of the verb 'holds' is 'cave' (inC), so it should be in the singular, i.e. 'holds'. The remaining two sentences –C and E –are fully correct. Hence, CE.

Question: 69The sentences given in each question, when properly sequenced, form a coherent paragraph. Each sentence is labelled with a

letter. Enter in the box provided below the most logical order of sentences to construct a coherent paragraph.Note: Use the virtual keyboard provided to enter your answer.A. Yet one cannot really understand what philosophy means today simply by knowing this etymological fact.B. The etymological fallacy is the mistake of thinking that one can best understand what a word means by understanding its origin.C. Likewise, if you go into an Italian restaurant knowing only that 'tagliatelle' literally means 'little boot laces', you won't have much ideawhat you're going to end up eating if you order it.D. But this is evidently not always true.

1) ABE

2) BCE

3) CDE

4) AEB

1) CE

2) BA

3) AC

4) BC

Login ID:51A07053/Student Name:ANAND RAJ/Overall Score:0

© CopyrightIMS Learning Resources Pvt.Ltd.,Mumbai.All copyrights to this material vestswith IMS Learning Resources Pvt.Ltd.No part of this materials either in part oras a whole shall be copied,printed,electronically reproduced,sold or distributed without the writtenconsent of IMS Learing Resources Pvt.Ltd.and any such violation would entail initiation of suitable legal proceedings.

Page 63: Login ID:51A07053/Student Name:ANAND RAJ/Overall Score:0 · (i n sq. cm) of such a triangle? Explanation: Question: 9 Answer the question independently. Eight colleagues, four men

E. For example, the etymology of 'philosophy' is the Greek for 'love of wisdom'.

[quizky-text]

Explanation:B introduces the topic of the paragraph –the etymological fallacy –so it is clearly the first sentence in this sequence. The word 'philosophy'

present in E and A points to an EA link: E states the etymology of this word, and A points out that one cannot understand its currentmeaning based on this etymology. 'Likewise' in C links it to EA, since C provides a similar example. If D were placed after EAC, it wouldstart a new paragraph, not conclude this one. It fits best after B, as D states that the idea that 'one can best understand what a wordmeans by understanding its origin' is 'not always true'; then EAC follow from this, by providing examples of such cases. Hence, BDEAC.

Question: 70The sentences given in each question, when properly sequenced, form a coherent paragraph. Each sentence is labelled with a

letter. Enter in the box provided below the most logical order of sentences to construct a coherent paragraph.Note:Use the virtual keyboard provided to enter your answer.A. The strands are stuck together by a kind of zip of hydrogen bonds between the bases on one strand and those on the other.

B. So the twin helices of DNA have complementary sequences: wherever an A appears in one strand, a T appears in the other, and soforth.

C. DNA is a double-stranded polymer: two chains are twisted around one another in a double helix.

D. Although all the bases can form hydrogen bonds, they have distinct preferences in their pairing relationships: A sticks to T, and G to C.E. Each of these strands consists of a string of nucleotides, embodying encoded information.

[quizky-text]

Explanation:C, which is the only standalone sentence and which introduces the topic of the paragraph (DNA), has to be the first sentence. 'These

strands' in E links to the double strands of DNA mentioned in C, so E must follow from C. A continues the description of these strands. Dprovides further information about the 'hydrogen bonds' mentioned in A. The letters in B (A and T) are the ones mentioned in D, so Bfollows D. Thus the correct sequence is CEADB. Hence, CEADB.

Question: 71The passage given below is followed by a set of questions. Choose the most appropriate answer to each question.

Fashion is occasionally cast as art, but this is problematic. Some designers have appropriated aspects of art practice in their own work,but they remain within the structure of the fashion industry and use these borrowed methods to explore the nature of fashion itself. When,for example, in their early career, designers Viktor and Rolf decided just to stage fashion shows rather than produce any saleable clothes,their designs became one-offs, rare pieces that existed only as comments on the role of the show within the fashion system, rather thanwearable garments. However, their work remained within the context of the fashion world, discussed and reviewed by fashion journalists.Viktor and Rolf's interpretation of fashion incorporated a fascination with the role of the show, and its potential to test the boundaries of

1) BDEAC

2) CDEAB

3) DBAEC

4) ABDCE

1) CEADB

2) ADEBC

3) BCADE

4) ABCDE

Login ID:51A07053/Student Name:ANAND RAJ/Overall Score:0

© CopyrightIMS Learning Resources Pvt.Ltd.,Mumbai.All copyrights to this material vestswith IMS Learning Resources Pvt.Ltd.No part of this materials either in part oras a whole shall be copied,printed,electronically reproduced,sold or distributed without the writtenconsent of IMS Learing Resources Pvt.Ltd.and any such violation would entail initiation of suitable legal proceedings.

Page 64: Login ID:51A07053/Student Name:ANAND RAJ/Overall Score:0 · (i n sq. cm) of such a triangle? Explanation: Question: 9 Answer the question independently. Eight colleagues, four men

spectacle and display. They slip between art, theatre and film in the staging of their collections. For example, for autumn/winter 2000, thedesigners slowly dressed a single model in layer upon layer of garments, until she wore the whole collection. This commented on theprocess of fitting clothes on the body, which lies at the core of traditional fashion design.In Viktor and Rolf's designs and presentations, artistic methods are used to comment on the practice of fashion, but this does notnecessarily turn their fashion into art. Their work is shown in the context of the international fashion weeks, it is directed to a fashionaudience, and addresses the way clothing and body interact. Even when they were not putting their clothing into production, they followedthe fashion seasons, and importantly, they adhered to the fundamental elements of fashion: fabric and body.Fashion is sometimes compared to art in order to give it greater validity, depth and purpose. However, this perhaps reveals more aboutWestern concern that fashion lacks these qualities than it does about fashion's actual significance. A Balenciaga dress from the 1950s,when displayed in a pristine glass case in a gallery, may appear like a work of art. However, it does not need to be described as such inorder to convey its value or the skill that went into its creation. Like other design forms, such as architecture, fashion has its own particularconcerns that prevent it from ever being purely art, craft or industrial design. It is, rather, a three-dimensional design form that incorporateselements of all these approaches. It is Balenciaga's exacting eye for precise form that brings balance and drama to the drape andstructure of the fabric, combined with the craft skill of his atelier workers, that turns it into an exceptional piece of fashion clothing. It doesnot need to be called art in order to validate its status, and this term ignores the reason, beyond his desire to create and test theparameters of fashion design, that Balenciaga's dresses were brought into being: to clothe a woman, and, ultimately, to sell more designs.This should not be seen to diminish his achievement, but to help to understand the way he has worked to exploit these 'limitations' tocreate fashions that can inspire the viewer as much as the wearer.Fashion should be understood on its own terms, and this makes its interactions with other aspects of art and culture more interesting. Itopens up the way art, design and commerce connect and overlap in some practitioners' work. Indeed, one of the things that makesfashion so fascinating, and for some, so problematic, is the fact that it continually appropriates, reconfigures and tests the boundaries ofthese definitions. Thus, fashion can highlight tensions concerning what is valued in a culture. Diverse designers and artists have producedwork that played upon cultural contradictions and attitudes. In fashion's case, focus on body and cloth, and the fact that it is, usually,designed to be worn and sold, distinguishes it from fine art. However, this does not prevent fashion from being meaningful, and the artworld's continued fascination with fashion underlines its cultural significance.

What is the main question the author is trying to answer in this passage?

Explanation:The question in [4] is barely answered at all in this passage, apart from the example of Viktor and Rolf's fashion shows. While the

question in [3] is answered through the same example or through the example of the Balenciaga dress, as well as in the last paragraph, itis not the main question being asked throughout the passage. In this passage, the author mainly tries to show that fashion is not art,though the two may have some things in common. Thus, [2] is irrelevant, while [1] is the main question she is trying to answer. Hence, [1].

Question: 72The passage given below is followed by a set of questions. Choose the most appropriate answer to each question.

Which of these statements is/are true about Viktor and Rolf's work, as per this passage? Choose as many statements as apply.Note: Your answer should be in alphabetical order. Use the virtual keyboard provided to enter your answer.

A.They no longer produce wearable clothes.B.The fashion shows that they stage involve elements of art and film.C.They use their fashion shows to comment on the practice of fashion.D.They view their fashion shows as expressions of art rather than fashion.E. Their work tends to be out of touch with the fundamental elements of fashion.

1) Is fashion art?

2) What is the artistic merit of fashion?

3) What do art and fashion have in common?

4) How did designers turn fashion into art?

Login ID:51A07053/Student Name:ANAND RAJ/Overall Score:0

© CopyrightIMS Learning Resources Pvt.Ltd.,Mumbai.All copyrights to this material vestswith IMS Learning Resources Pvt.Ltd.No part of this materials either in part oras a whole shall be copied,printed,electronically reproduced,sold or distributed without the writtenconsent of IMS Learing Resources Pvt.Ltd.and any such violation would entail initiation of suitable legal proceedings.

Page 65: Login ID:51A07053/Student Name:ANAND RAJ/Overall Score:0 · (i n sq. cm) of such a triangle? Explanation: Question: 9 Answer the question independently. Eight colleagues, four men

[quizky-text]

Explanation:According to the first paragraph, Viktor and Rolf did not produce wearable clothes early in their career; whether they still follow this

practice or not is not clear (though the last sentence of paragraph 3 suggests otherwise). So it cannot be said that they 'no longer' producewearable clothes. Thus A is wrong. B is correct, as it is stated in the second sentence of paragraph 2. C is also correct, as it is stated inthe first sentence of paragraph 3. The passage does not clarify how Viktor and Rolf see their fashion shows, but the third paragraphsuggests that they see them in terms of fashion rather than art. So D is wrong. E is incorrect as well, as it contradicts the last sentence ofparagraph 3. Therefore, only B and C are true about Viktor and Rolf's work. Hence, BC.

Question: 73The passage given below is followed by a set of questions. Choose the most appropriate answer to each question.

What is the author's attitude towards the vintage Balenciaga dress?

Explanation:Refer to paragraph 4. The author does not deny the similarities that a work of fashion like a vintage Balenciaga dress may have to art;

rather, she says that, like other design forms such as architecture, it is not purely art. So [2] is a misinterpretation of her point. The authorstates that 'fashion has its own particular concerns that prevent it from ever being purely art, craft or industrial design', so [3] is amisinterpretation as well. There is no mention of the 'beauty' of the dress; also, the author points out that the dress does not need to beseen as a work of art in order for its unique qualities as a work of fashion to be appreciated. So [1] is incorrect, and [4] best encapsulatesthe author's attitude towards the vintage Balenciaga dress. Hence, [4].

Question: 74The passage given below is followed by a set of questions. Choose the most appropriate answer to each question.

In the author's opinion, should fashion be considered art?

Explanation:Throughout this passage, the author argues against the idea of considering fashion as art. While she states that fashion is often

compared to art, she herself does not think the comparison is necessary or useful (see paragraph 4). So both [1] and [2], which answerthe question positively, can be ruled out. On the other hand, in the last paragraph, the author acknowledges that fashion does overlap withart somewhat and that the two are culturally connected, so [4], which claims that the two are completely separate, is too extreme. Theauthor's view is best encapsulated by [3]. Hence, [3].

Question: 75

1) BC

2) AC

3) DA

4) AB

1) She thinks that it is beautiful and a good example of the artistic aspects of fashion.

2) She thinks that it has similarities to other design forms like architecture, but not to art.

3) She considers it an example of craft or industrial design that transcends its limitations.

4) She considers it a highly admirable work of fashion, but not necessarily a work of art.

1) Yes – because seeing fashion in terms of art gives it greater validity, depth and purpose.

2)Yes –even though the relationship between fashion and art is problematic, fashion continually appropriates, reconfigures

and tests the boundaries between the two.

3) No –fashion should be understood on its own terms, though it does overlap with art to some extent.

4) No –the two are completely separate entities and fashion does not need to be called art in order to validate its status.

Login ID:51A07053/Student Name:ANAND RAJ/Overall Score:0

© CopyrightIMS Learning Resources Pvt.Ltd.,Mumbai.All copyrights to this material vestswith IMS Learning Resources Pvt.Ltd.No part of this materials either in part oras a whole shall be copied,printed,electronically reproduced,sold or distributed without the writtenconsent of IMS Learing Resources Pvt.Ltd.and any such violation would entail initiation of suitable legal proceedings.

Page 66: Login ID:51A07053/Student Name:ANAND RAJ/Overall Score:0 · (i n sq. cm) of such a triangle? Explanation: Question: 9 Answer the question independently. Eight colleagues, four men

In the following questions, a verb is given followed by multiple sentences in which it is used as a phrasal verb. Identify thesentences in which the phrasal verb is used incorrectly or inappropriately. Enter in the box provided below the letterscorresponding only to the sentences in which the usage of the word is incorrect or inappropriate, in alphabetical order.Note: Use the virtual keyboard provided to enter your answer.-texMAKE

A. We decided to make to with what little we had.B. Her father plans to make over the family business to her.C. The thieves made of with lakhs of rupees worth of jewellery.D. The newspapers did not print the article, as they thought it did not make for interesting news.E. The workmen were asked to work longer hours to make out for the time lost due to the recent rains.[quizky-text]

Explanation:There is no phrasal verb such as 'make to with'; the correct verb in A should be to 'make do with', meaning to 'manage with limited

means'. In B, 'make over' is correctly used to mean to 'transfer (property)'. C is again incorrect: the correct phrasal verb should be to 'makeoff with', meaning to 'steal'. 'Make for' in D correctly means to 'tend to result in' or 'be received as'. In E, the correct phrasal verb should be'make up for', meaning 'compensate for', not 'make out for'. Thus, only B and D are correct, and A, C and E involve incorrect usage ofphrasal verbs. Hence, ACE.

Question: 76In the following questions, a verb is given followed by multiple sentences in which it is used as a phrasal verb. Identify the

sentences in which the phrasal verb is used incorrectly or inappropriately. Enter in the box provided below the letterscorresponding only to the sentences in which the usage of the word is incorrect or inappropriate, in alphabetical order.Note:Use the virtual keyboard provided to enter your answer.STAND

A. Please do come in – don't stand on ceremony.B. Their political party stands for religious freedom.C. She is so tall that she easily stands out in a group.D. The teacher told the students to stand with respectfully when the principal came in.E. If you stand to your problems, they may not seem so difficult.F. He decided to stand down as the chairman of the board.[quizky-text]

Explanation:In A, 'stand on' correctly means to 'be particular about'. 'Stand for' in B means to 'advocate'. 'Stand out' in C means to 'be prominent'.

'Stand down' in F means to 'step aside' or 'retire'. So these four sentences are examples of correct usage. On the other hand, 'stand with'is meaningless; the correct phrasal verb in D should be 'stand up'. In E, the correct phrasal verb should be 'stand up to', meaning to 'dealwith' or 'confront', not 'stand to'. Thus, D and E are examples of incorrect usage. Hence, DE.

1) ACE

2) CAE

3) ADE

4) DBA

1) DE

2) AD

3) BC

4) AB

Login ID:51A07053/Student Name:ANAND RAJ/Overall Score:0

© CopyrightIMS Learning Resources Pvt.Ltd.,Mumbai.All copyrights to this material vestswith IMS Learning Resources Pvt.Ltd.No part of this materials either in part oras a whole shall be copied,printed,electronically reproduced,sold or distributed without the writtenconsent of IMS Learing Resources Pvt.Ltd.and any such violation would entail initiation of suitable legal proceedings.

Page 67: Login ID:51A07053/Student Name:ANAND RAJ/Overall Score:0 · (i n sq. cm) of such a triangle? Explanation: Question: 9 Answer the question independently. Eight colleagues, four men

Question: 77Three out of four sentences in the options, when correctly sequenced, form a coherent paragraph. Which of the following

sentences does not fit into the context?

Explanation:All four sentences are about the climate of Puerto Rico. However, while three of the sentences are about cyclones and hurricanesexperienced by Puerto Rico, sentence [1] is about its general climate type and the temperatures it experiences. So [1] does not fit into thesequence formed by the other three sentences, which can be logically arranged as: [3]-[4]-[2]. Hence, [1].

Question: 78Three out of four sentences in the options, when correctly sequenced, form a coherent paragraph. Which of the following

sentences does not fit into the context?

Explanation:Sentence [2] chronologically begins this sequence, as it introduces the topic of a photo of Earth taken from space, before spaceflight evenbegan. [4] talks about when the prediction came true, and [1] elaborates on this point. So these three sentences are clearly linked to eachother in the sequence [2]-[4]-[1]. On the other hand, though [3] does make a reference to the iconic image of the Earth seen from space, itis about a different topic altogether, viz. the beginning of the global environmental movement, so it does not fit into the sequence. Hence,[3].

Question: 79The passage given below is followed by a set of questions. Choose the most appropriate answer to each question.If we could journey back into the past, history would be impossible to write. As soon as a historian recorded the history of the past,someone could go back into the past and rewrite it. Not only would time machines put historians out of business, but they would enable usto alter the course of time at will. If, for example, we were to go back to the era of the dinosaurs and accidentally step on a mammal thathappens to be our ancestor, perhaps we would accidentally wipe out the entire human race. History would become an unending, madcapMonty Python episode, as tourists from the future trampled over historic events while trying to get the best camera angle.Time travel poses all sorts of problems, both technical as well as social. The moral, legal and ethical issues are raised by Larry Dwyer,who notes, 'Should a time traveller who punches his younger self (or vice versa) be charged with assault? Should the time traveller whomurders someone and then flees into the past for sanctuary be tried in the past for crimes he committed in the future? If he marries in thepast can he be tried for bigamy even though his other wife will not be born for almost 5,000 years?'But perhaps the thorniest problems are the logical paradoxes raised by time travel. For example, what happens if we kill our parentsbefore we are born? This is a logical impossibility. It is sometimes called the 'grandfather paradox'.

1) The climate of Puerto Rico falls into the tropical climatic zone, with moderate temperatures the year round.

2)On average, a quarter of Puerto Rico's annual rainfall comes from tropical cyclones, which are more prevalent duringperiods of La Niñathan El Niño.

3)Puerto Rico experiences the Atlantic hurricane season, similar to the remainder of the Caribbean Sea and North Atlanticoceans.

4)A cyclone of tropical storm strength passes near Puerto Rico, on average, every five years, and a hurricane, on average,every seven years.

1)It's not that those first views told us much we didn't already know about the Earth, but they gave us an icon and anemotional experience of the beauty and seeming fragility of our world that has been with us ever since.

2)'Once a photograph of the Earth, taken from the outside, is available, a new idea as powerful as any in history will be letloose,' predicted astronomer Sir Fred Hoyle in 1948, a decade before the dawn of space flight.

3)It was around the same time, and with the same icon of that small blue jewel we call Earth floating in the blackness of

space, that a global environmental movement began to form.

4)When unmanned rockets took the first pictures of the Earth from outside, and when the first generation of astronauts saw

for themselves our world in its entirety, the prediction came true.

Login ID:51A07053/Student Name:ANAND RAJ/Overall Score:0

© CopyrightIMS Learning Resources Pvt.Ltd.,Mumbai.All copyrights to this material vestswith IMS Learning Resources Pvt.Ltd.No part of this materials either in part oras a whole shall be copied,printed,electronically reproduced,sold or distributed without the writtenconsent of IMS Learing Resources Pvt.Ltd.and any such violation would entail initiation of suitable legal proceedings.

Page 68: Login ID:51A07053/Student Name:ANAND RAJ/Overall Score:0 · (i n sq. cm) of such a triangle? Explanation: Question: 9 Answer the question independently. Eight colleagues, four men

There are three ways to resolve these paradoxes. First, perhaps you simply repeat past history when you go back in time, thereforefulfilling the past. In this case, you have no free will. You are forced to complete the past as it was written. Thus, if you go back into thepast to give the secret of time travel to your younger self, then it was meant to happen that way. The secret of time travel came from thefuture. It was destiny. (But this does not tell us where the original idea came from.)Second, you have free will, so you can change the past, but within limits. Your free will is not allowed to create a time paradox. Wheneveryou try to kill your parents before you are born, a mysterious force prevents you from pulling the trigger. This position has been advocatedby the Russian physicist Igor Novikov. (He argues that there is a law preventing us from walking on the ceiling, although we might want to.Hence there might be a law preventing us from killing our parents before we are born. Some strange law prevents us from pulling thetrigger.)Third, the universe splits into two universes. This possibility seems to be the one consistent with the quantum theory and the idea of themultiverse. The idea is that when you try to go into the past, the time line of our universe branches off, creating a parallel universe. Thus,whenever we go back into the river of time, the river forks into two rivers, and one time line becomes two time lines, or what is called the'many worlds' approach.This means that all time travel paradoxes can be solved. If you have killed your parents before you were born, it simply means you havekilled some people who are genetically identical to your parents, with the same memories and personalities, but they are not your trueparents.

Choose a suitable title for this passage.

Explanation:The 'grandfather paradox' is only one of the problems caused by time travel mentioned in this passage, so [1] is not a comprehensive

title. While the author does suggest a view in the last two paragraphs that would help resolve similar time paradoxes, the moral, legal andethical issues posed by time travel (mentioned in paragraph 2) are left unresolved. So [2] is not a suitable title either. [4] can be ruled out,as parallel universes are mentioned only in the last two paragraphs. The passage is mostly about a number of dilemmas raised by theidea of time travel, both technical and social ones. Hence, [3].

Question: 80The passage given below is followed by a set of questions. Choose the most appropriate answer to each question.

Which of these statements is/are implicit in the 'parallel universe' approach to resolving time paradoxes? Choose as many statements asapply.

Note: Your answer should be in alphabetical order. Use the virtual keyboard provided to enter your answer.

A. You can make all the changes you want to time, without worrying about the consequences.B. If you go back in time, you automatically wind up in a timeline that is not the one you have actually experienced.C. The issue of paradoxes does not arise at all, because all changes to time would occur in a different timeline.

[quizky-text]

Explanation:

1) The 'Grandfather Paradox'

2) Resolving Time Paradoxes

3) Dilemmas raised by Time Travel

4) Time Travel in Parallel Universes

1) BC

2) AB

3) BAC

4) ABC

Login ID:51A07053/Student Name:ANAND RAJ/Overall Score:0

© CopyrightIMS Learning Resources Pvt.Ltd.,Mumbai.All copyrights to this material vestswith IMS Learning Resources Pvt.Ltd.No part of this materials either in part oras a whole shall be copied,printed,electronically reproduced,sold or distributed without the writtenconsent of IMS Learing Resources Pvt.Ltd.and any such violation would entail initiation of suitable legal proceedings.

Page 69: Login ID:51A07053/Student Name:ANAND RAJ/Overall Score:0 · (i n sq. cm) of such a triangle? Explanation: Question: 9 Answer the question independently. Eight colleagues, four men

Refer to the penultimate paragraph: 'Thus, whenever we go back into the river of time, the river forks into two rivers, and one time linebecomes two time lines'. This implies that the mere act of going back into the past automatically gives rise to two separate timelines; andthe fact that no paradox occurs when you change the past implies that you always wind up in a different timeline when you go back to thepast. Thus B is correct. The example in the last paragraph suggests that the reason the paradoxes don't arise is because the changes youmake would be in a different timeline. So C is implicit as well. Only A is incorrect: just because no time paradoxes would occur, that doesnot mean that there would not be any negative consequences – if the changes you make involve murdering people, there would certainlybe moral, social and legal consequences. Therefore, only B and C are implicit. Hence, BC.

Question: 81The passage given below is followed by a set of questions. Choose the most appropriate answer to each question.

Assume that someday time travel becomes a reality, and time travellers find that they can go back into the past, but are able to changeonly certain aspects of the past. This would mean that:

Explanation:Refer to paragraph 4. The first method of resolving time paradoxes claims that time travellers can make no changes whatsoever in the

past. So clearly, this method is not the correct one, as the scenario in the question states that some changes can be made. However,there is no information as to which those changes are – are they ones that would cause a paradox or not? The second method talks onlyabout a prevention of changes that would cause paradoxes. So in the given scenario, it is likely that the second method is the correct one,but until we have more information regarding the changes in question, we cannot say with certainty that it is. Hence, [3].

Question: 82The passage given below is followed by a set of questions. Choose the most appropriate answer to each question.

The author of this passage is least likely to be a:

Explanation:In discussing the resolution of time paradoxes, the author brings up ideas from physics – for example, the Russian physicist Igor

Novikov's idea in paragraph 5, or the parallel universe idea in paragraph 6. So it is possible that he might be a physicist. It is quite likelythat he may be a science fiction writer, exploring aspects of time travel to be used in his books. He could be a philosopher, exploring themoral, legal, ethical and logical dilemmas raised by the concept of time travel. But while the author mentions the negative effect that timetravel would have on history, he does not discuss any actual history. So it is not very likely that he is a historian. Hence, [1].

Question: 83Each of the following questions has a sentence with two blanks. Given below each question are four pairs of words. Choose thepair that best completes the sentence.The ______ tale of Walcott's discovery of the Burgess Shale fossil site has it that his mule lost a shoe at the critical spot, but since he wasalready prospecting for fossils in those mountains, any intervention by ______ was mediated by design.

1) The first method of resolving time paradoxes is the correct one.

2) The first method of resolving time paradoxes may be the correct one.

3) The second method of resolving time paradoxes may be the correct one.

4) The second method of resolving time paradoxes is the correct one.

1) Historian.

2) Physicist.

3) Philosopher.

4) Science fiction writer.

1) esoteric ... tenacity

2) apocryphal ... serendipity

Login ID:51A07053/Student Name:ANAND RAJ/Overall Score:0

© CopyrightIMS Learning Resources Pvt.Ltd.,Mumbai.All copyrights to this material vestswith IMS Learning Resources Pvt.Ltd.No part of this materials either in part oras a whole shall be copied,printed,electronically reproduced,sold or distributed without the writtenconsent of IMS Learing Resources Pvt.Ltd.and any such violation would entail initiation of suitable legal proceedings.

Page 70: Login ID:51A07053/Student Name:ANAND RAJ/Overall Score:0 · (i n sq. cm) of such a triangle? Explanation: Question: 9 Answer the question independently. Eight colleagues, four men

Explanation:The 'tale', as given in this sentence, cannot be described as 'iconoclastic', which means 'attacking cherished beliefs or traditions', so [4]can be eliminated. The tale is about a chance discovery; the author then goes on to say that some 'design', i.e. purpose or intention (onthe discoverer's part), was also involved. Thus, the second blank should have a word that means roughly the same as 'chance'. Only'serendipity', meaning 'good fortune', fits this criterion. 'Apocryphal', meaning 'of doubtful authenticity', also fits into the first blank, as it fitsthe sceptical tone of the rest of the sentence. Hence, [2].

Question: 84Each of the following questions has a sentence with two blanks. Given below each question are four pairs of words. Choose thepair that best completes the sentence.Faraday was born to working-class parents and ______ out a meagre existence as a/an ______ bookbinder in the early 1800s.

Explanation:'Spun out' means 'went out of control', which does not fit into this context at all. 'Wrested' cannot be followed by 'out'. Both 'wrung' and'eked' when followed by 'out' mean 'get by effort', which correctly fits in the first blank. However, the collocation 'aficionado bookbinder' isincorrect (the correct form would be 'bookbinding aficionado'). On the other hand, 'apprentice' (meaning 'someone who works for a skilledperson to learn a trade') fits both grammatically and semantically in the second blank. Hence, [4].

Question: 85Four alternative summaries are given below each text. Choose the option that best captures the essence of the text.

I do not think it likely that the human mind (let alone the wonders it concocts) will ever be explained in molecular terms, any more thanKing Lear is explained by the alphabet. Most scientists do not believe so either. Phenomena are hierarchical: all things cannot beunderstood by considering only what transpires on a single rung. No matter how well I understand the way a transistor works, I will not beable to deduce from this knowledge why my computer crashes. If I sow seeds that fail to grow, I will do better to begin by thinking aboutthe nutrient content, humidity and temperature of my soil than by performing a genetic analysis of the seeds.

Explanation:The paragraph is specifically about human minds, not human beings as a whole, so [1] can be eliminated as once. [3] reads more like a

definition of the concept of the hierarchy of phenomena than a summary of the point made in this paragraph. The topic of this paragraph ishuman minds; the other items, such as computers and seeds, are mentioned only as examples to illustrate the idea that phenomena arehierarchical. So [4], which treats the main topic as equivalent to the examples, is incorrect as well. The concept the author is trying toexplain is that phenomena are hierarchical, and thus a phenomenon on one level of the hierarchy cannot be understood in terms of itscomponents lower in the hierarchy; the examples serve to make the same point. Only [2] is therefore a fully correct summary. Hence, [2].

Question: 86

3) irrefutable ... persistence

4) iconoclastic ... providence

1) spun ... tyro

2) wrung ... aficionado

3) wrested ... veteran

4) eked ... apprentice

1) Given that phenomena are hierarchical, human beings cannot be understood in terms of their molecular make-up.

2)Going by the principle that phenomena are hierarchical, it is unlikely that the human mind will ever be understood in termsof its component molecules.

3)The concept of the hierarchy of phenomena means that things higher in the hierarchy, such as human minds, cannot be

understood in terms of lower ones, such as molecules.

4)The concept of the hierarchy of phenomena suggests that things like human minds, computers, seeds, etc. cannot be

understood in terms of their components.

Login ID:51A07053/Student Name:ANAND RAJ/Overall Score:0

© CopyrightIMS Learning Resources Pvt.Ltd.,Mumbai.All copyrights to this material vestswith IMS Learning Resources Pvt.Ltd.No part of this materials either in part oras a whole shall be copied,printed,electronically reproduced,sold or distributed without the writtenconsent of IMS Learing Resources Pvt.Ltd.and any such violation would entail initiation of suitable legal proceedings.

Page 71: Login ID:51A07053/Student Name:ANAND RAJ/Overall Score:0 · (i n sq. cm) of such a triangle? Explanation: Question: 9 Answer the question independently. Eight colleagues, four men

Four alternative summaries are given below each text. Choose the option that best captures the essence of the text.

Have you ever noticed how the worst part of someone's personality is often also the best? You may know an anally retentive, detail-oriented accountant who never cracks a joke, nor understands any, but this is in fact what makes him the perfect accountant. Or you mayhave a flamboyant aunt who constantly embarrasses everyone with her big mouth, yet is the life of every party. The same duality appliesto our species. We human beings certainly don't like our aggressiveness –at least on most days –but would a society without it be sogreat? Wouldn't we all be as meek as lambs? Our sports teams wouldn't care about winning or losing, entrepreneurs would be impossibleto find, and pop stars would sing only boring lullabies. I'm not saying that aggressiveness is good, but it enters into everything we do, notjust murder and mayhem.

Explanation:As per the first sentence of the paragraph, the worst and best of individuals' personalities can often be the one and the same traits. So [1]

and [2], which imply that the worst and best are separate traits, are incorrect readings of the paragraph. The author does not consideraggressiveness to be the 'best feature' of the human species –he just suggests that it has some positive aspects. So [4] is incorrect, and[3] is the best summary. Hence, [3].

Question: 87The passage given below is followed by a set of questions. Choose the most appropriate answer to each question.

'You don't happen to know someone who ...' the headhunter murmurs before describing a candidate oddly like yourself. For many, the oldheadhunting cliches are something of a joke. Yet the industry itself is anything but; last year its global revenues totalled an impressive$3.5 billion, according to the Economist Intelligence Unit, a business-information publisher. The EIU estimates that revenues for the topheadhunters (or executive-search firms, as they call themselves) rose by 35-40% last year. Not only is their business booming,headhunters have even acquired respectability, of a sort. Britain's royal family is said to employ them, while the BBC recently based a TVdrama series on the subject.Headhunters, like the management consultancies to which many trace their origins, have benefited from the current craze among firms to'outsource' many things they once did for themselves. But two other changes have also helped the executive searchers. As the settledmanagement patterns of seniority have crumbled, top managers are switching jobs much more rapidly. And as firms move into new, far-flung markets, they have often had to look beyond their ranks for managers, both local and expatriate, with specialized knowledge. Arecent survey by Amrop International, a headhunter, and Harvard Business School, found that multinational companies expect 74% oftheir managers to be host-country nationals in five years' time, compared with 45% today.All this has given headhunters more work on both sides of the Atlantic. This year the American market, which generates nearly $2 billion –up from about $100 million in 1970 – should grow by 10-12%. In Europe, revenues may grow by around 30% this year.Like accountants, who ran into trouble by peddling consultancy services to audit clients, the main challenge for the recruitment business isto avoid conflicts of interest. The first headhunting firm was set up in the United States in 1926 by Thorndike Deland. Then themanagement consultancies stepped in. Two of the top four firms, Heidrick & Struggles and Spencer Stuart, were set up by formerconsultants from Booz Allen & Hamilton. But clients feared the consultants would suggest the creation of new jobs in order to be able tocharge for filling them. So recruitment was spun off into a separate business, though the searchers still like to call themselves'consultants'.Now such conflicts centre on how headhunters are paid. Clients normally turn to headhunters for jobs with salaries of $110,000 or more,which require discreet approaches, rather than an open advertisement or the use of the old-boy network. Typically, the search firm wouldearn a commission of one third of the successful candidate's initial compensation package (this can be as much as $750,000 for a really

1)The negative sides of individuals' personalities can hide much more positive qualities; similarly, a negative trait like

aggressiveness can be beneficial to the human species.

2)Just as individuals need the negative aspects of their personalities to bring out their positive sides, human beings need a

negative trait like aggressiveness to bring out their positive qualities.

3)Just as the worst aspects of individuals' personalities can often also be the best, aggressiveness, a negative aspect of the

human species, also has positive aspects.

4)The worst parts of individuals' personalities can sometimes also be the best; similarly, the worst part of the human species

–aggressiveness –is also its best feature.

Login ID:51A07053/Student Name:ANAND RAJ/Overall Score:0

© CopyrightIMS Learning Resources Pvt.Ltd.,Mumbai.All copyrights to this material vestswith IMS Learning Resources Pvt.Ltd.No part of this materials either in part oras a whole shall be copied,printed,electronically reproduced,sold or distributed without the writtenconsent of IMS Learing Resources Pvt.Ltd.and any such violation would entail initiation of suitable legal proceedings.

Page 72: Login ID:51A07053/Student Name:ANAND RAJ/Overall Score:0 · (i n sq. cm) of such a triangle? Explanation: Question: 9 Answer the question independently. Eight colleagues, four men

top job). But since that encourages recruiters to search for expensive candidates, the trend is towards fixed fees, says Nancy GarrrisonJenn, an industry analyst.The business has also become more professional. In Europe, gone are the days of the gin-and-tonic brigade of failed bankers. At EgonZehnder International (EZI), a Swiss search firm, consultants need an MBA or a second degree. In the 1980s, EZI was one of the topthree destinations for former staff from McKinsey, a management consultancy. Not surprisingly, EZI headhunts its own staff – andsuccessfully too: its staff turnover over the past 30 years is just 2%, compared with an average for the industry of 30%. EZI says thataspiring consultants who send in their CVs 'are probably not the right sort of person for us'.

Which of the following best states the central idea of this passage?

Explanation:This passage focuses on the present status of the headhunting business, and its origins. There is not much said about its future; and

there is certainly nothing specifically stated about its prospective growth in the next two years. So [3] can be ruled out immediately. Thephrasing of [1] is ambiguous: it could wrongly imply that the headhunting business separated from consultancy work just last year. So [1]is incorrect as well. The origins of the industry described in this passage do not seem particularly 'humble'. So [2] is not quite right either.Only [4] correctly states the central idea of this passage. Hence, [4].

Question: 88The passage given below is followed by a set of questions. Choose the most appropriate answer to each question.

Which of the following is one of the pioneering headhunting firms, as per this passage?

Explanation:According to paragraph 2, Amrop International is a current headhunting firm; whether it was also one of the pioneering ones is not stated

in the passage. As per paragraph 4, 'The first headhunting firm was set up in the United States in 1926 by Thorndike Deland'; whether thatwas also the name of the firm is not clear, so [1] is not quite correct. Spencer Stuart was one of the pioneering headhunting firms set up byformer consultants from Booz Allen & Hamilton; so [4] is correct, while [3] is not. Hence, [4].

Question: 89The passage given below is followed by a set of questions. Choose the most appropriate answer to each question.

Which of the following is not one of the factors that helped headhunters to grow substantially?

1)After separating from consultancy work in USA, the headhunting business performed impressively last year, and the future

holds still better prospects for the industry.

2)Despite its humble beginnings in the consultancy industry in USA, the headhunting business has been a great success all

over the world, and is now extremely professional.

3)Headhunting, which was earlier a part of the business consultancy in USA, performed impressively worldwide last year and

is poised for further growth in the next two years.

4)Headhunting, which had its beginnings as a part of consultancy work in USA, is now a separate and more professional

business, registering an impressive growth globally.

1) Thorndike Deland

2) Amrop International

3) Booz Allen & Hamilton

4) Spencer Stuart

1) The current craze among firms to outsource many things they once did for themselves

2) Firms having to look for managers with knowledge of local management styles in new, far-flung markets

3) Top managers switching jobs much faster, as established patterns of secure jobs and promotions by seniority crumble

4) Globalization of business causing companies to look beyond their own ranks for managers with specialized knowledge

Login ID:51A07053/Student Name:ANAND RAJ/Overall Score:0

© CopyrightIMS Learning Resources Pvt.Ltd.,Mumbai.All copyrights to this material vestswith IMS Learning Resources Pvt.Ltd.No part of this materials either in part oras a whole shall be copied,printed,electronically reproduced,sold or distributed without the writtenconsent of IMS Learing Resources Pvt.Ltd.and any such violation would entail initiation of suitable legal proceedings.

Page 73: Login ID:51A07053/Student Name:ANAND RAJ/Overall Score:0 · (i n sq. cm) of such a triangle? Explanation: Question: 9 Answer the question independently. Eight colleagues, four men

Explanation:Refer to paragraph 2. Options [1], [3] and [4] are clearly stated as factors that have helped the growth of the headhunting industry. While

it is possible that the 'specialized knowledge' that companies look for among local managers in new, far-flung markets involves 'knowledgeof local management styles', this is not stated in the passage, and nor is it specifically inferable. Hence, [2].

Question: 90The passage given below is followed by a set of questions. Choose the most appropriate answer to each question.

Which of the following does not show the impressive growth achieved by the headhunting industry, as mentioned in the passage?

Explanation:Refer to the first paragraph. Options [2], [3] and [4] are clearly mentioned as indicators of the impressive growth achieved by the

headhunting industry. On the other hand, [1] is an incorrect reading of paragraph 3, which mentions that the American market is expectedto grow by 10-12%, whereas European revenues may grow by around 30%. Hence, [1].

Question: 91The passage given below is followed by a set of questions. Choose the most appropriate answer to each question.

In the early stages of its development, the headhunting business in USA was a part of the consultancy work, but was later spun off into aseparate business because:

Explanation:Refer to paragraph 4. The example of the accountants is mentioned only as an analogy, not an explanation for why headhunting became

a separate business. So [1] is incorrect. There is no mention of [3] anywhere in the passage. [4] is mentioned in paragraph 5, butregarding a different issue (how headhunters are paid), so it is irrelevant. Only [2], which is stated in paragraph 4, is the correctexplanation for why the headhunting business in USA branched off from consultancy work to form a separate business. Hence, [2].

Question: 92The sentences given in each question, when properly sequenced, form a coherent paragraph. Each sentence is labelled with a

letter. Choose the most logical order of sentences, from among the given choices, to construct a coherent paragraph.A. Adults who share electronic texts with young children are often improving their own transliteracy as they do so (developing adult literacyis frequently an added-value aspect of children's literature).B. Once, literacy focused on the ability to read and write, primarily in fixed print and paper-based media.C. Encountering traditional texts in multiple versions - including multimedia - is an effective way to encourage both cultural and text-relatedliteracy.D. Children are more comfortable with transliteracy, whereas many adults, who have become transliterate long after first learning to read,may be less comfortable moving between media.E. In the 21st century, however, it is necessary to think in terms of transliteracy - literacy that crosses between media and is no longerexclusively text-based.

1) This year, the American market is expected to grow by 30%.

2) Revenues for top executive-search firms rose by 35-40% last year.

3) Global revenues for the headhunting industry totalled $3.5 billion last year.

4) The headhunting business has acquired respectability –even the British royal family employs it.

1) Accountants treated them as two separate business activities to avoid conflicts of interest.

2) Clients feared that consultants would suggest the creation of new jobs so that they could charge for filling them.

3) Focusing on recruitment would give it a speciality status and thus clients could be provided better service.

4)The commission-based compensation model of consultancy work led to recruiters searching for unnecessarily expensive

candidates.

1) ADBEC

Login ID:51A07053/Student Name:ANAND RAJ/Overall Score:0

© CopyrightIMS Learning Resources Pvt.Ltd.,Mumbai.All copyrights to this material vestswith IMS Learning Resources Pvt.Ltd.No part of this materials either in part oras a whole shall be copied,printed,electronically reproduced,sold or distributed without the writtenconsent of IMS Learing Resources Pvt.Ltd.and any such violation would entail initiation of suitable legal proceedings.

Page 74: Login ID:51A07053/Student Name:ANAND RAJ/Overall Score:0 · (i n sq. cm) of such a triangle? Explanation: Question: 9 Answer the question independently. Eight colleagues, four men

Explanation:E, which explains the term 'transliteracy', must come earlier in the sequence than A and D, both of which use this term. Only option [2] -

BECDA - fulfils this criterion. B makes a suitable opening statement, explaining the former focus of literacy; E introduces the currentconcept of 'transliteracy'; C expands on this concept; and D and A talk about children's and adults' relationship with transliteracy,respectively. Hence, [2].

Question: 93The sentences given in each question, when properly sequenced, form a coherent paragraph. Each sentence is labelled with a

letter. Choose the most logical order of sentences, from among the given choices, to construct a coherent paragraph.A. Certainly, there is no solid surface that a human could ever stand upon.B. Probably, the troposphere of each giant planet merges seamlessly into a fluid interior at temperatures and pressures so high that thereis no distinction between gas and liquid.C. These occur in the planet's troposphere, above which are largely transparent and progressively less dense layers classifiable in thesame way as for the Earth's atmosphere.D.By convention, the size of a giant planet is measured from the top of its clouds.E. The base of a giant planet's troposphere is hard to define and has never been explored even in the case of Jupiter, where in 1995 anentry probe released by the Galileo spacecraft reached a depth of 160 kilometres below the cloud-tops before pressure (22 atmospheres)and temperature (153 °C) put paid to it.

Explanation:Either D or E can be a valid opening sentence, so we need to look at internal links to identify the correct option. The DBC link in option [2]

has no logical flow: D talks of the top of the clouds in a planet, B talks about its interior, and C goes back to talking about the upper part ofthe atmosphere. So [2] can be eliminated. The AC link in [1] and [3] makes no sense, as there is nothing in A that could possibly be theantecedent of the pronoun 'these' in C. This rules out [1] and [3] as well. The DCEBA sequence in [4] makes the most sense: theparagraph explains that a giant planet is measured from the top of its clouds, because its base is hard to define as it is not solid. Hence,[4].

Question: 94In the following questions, there are sentences or parts of sentences that form a paragraph. Identify the sentence(s) or part(s)

of sentence(s) that is/are correct in terms of grammar and usage (including spelling, punctuation and logical consistency). Then,choose the most appropriate option.A. Perhaps the most familiar application of the principle that like produces like is the attempt which has been made by many peoples inmany agesB. to injure or destroy an enemy by injuring or destroying an image of him, in the belief that, just as the image suffers, as does the man,and that when it perishes he must die.C. A few instances out of many may be given to prove at once the wide diffusion of the practise over the world and its remarkablepersistence through the ages.D.Thousands of years ago it was known to the sorcerers of ancient India, Babylon and Egypt, as well as of Greece and Rome,E.and to this day it is still resorted by the indigenous peoples of in Australia, Africa and Scotland.

2) BECDA

3) CABED

4) DACEB

1) EBDAC

2) EADBC

3) DBACE

4) DCEBA

1) A & D

2) B, C & D

Login ID:51A07053/Student Name:ANAND RAJ/Overall Score:0

© CopyrightIMS Learning Resources Pvt.Ltd.,Mumbai.All copyrights to this material vestswith IMS Learning Resources Pvt.Ltd.No part of this materials either in part oras a whole shall be copied,printed,electronically reproduced,sold or distributed without the writtenconsent of IMS Learing Resources Pvt.Ltd.and any such violation would entail initiation of suitable legal proceedings.

Page 75: Login ID:51A07053/Student Name:ANAND RAJ/Overall Score:0 · (i n sq. cm) of such a triangle? Explanation: Question: 9 Answer the question independently. Eight colleagues, four men

Explanation:In B, the correct correlative conjunction pair should be 'just as ... so', not 'just as ... as', so B is incorrect. In C, the verb 'practise' is

incorrectly used instead of the noun 'practice'. The preposition 'to' is missing after 'resorted' in E. Therefore, only A and D are fullygrammatically correct. Hence, [1].

Question: 95In the following questions, there are sentences or parts of sentences that form a paragraph. Identify the sentence(s) or part(s)

of sentence(s) that is/are correct in terms of grammar and usage (including spelling, punctuation and logical consistency). Then,choose the most appropriate option.A. Our species has a distinctly subversive streak that ensures that, however much we look up to those in power, we're always happy totake them down a point.B.Present-day egalitarians, who range from hunter-gatherers to horticulturalists, show the same tendency.C.They emphasize sharing and suppress distinctions of wealth and power.D.The would-be chief who gets it into his head that he can order others around is openly told how amusing he is.E.Leaders who become bullies, are self-aggrandizing, fail to redistribute goods or deal with outside threats, quickly lose the respect andsupport of the community.

Explanation:In A, the correct idiomatic phrase should be to 'take someone down a peg/notch' (meaning to 'reduce someone's pride') not 'point'. There

is a parallelism error in E: the first three items in the list have their own verbs ('become', 'are' and 'fail', respectively), but the last item ('dealwith outside threats') uses the same verb as the preceding item – i.e. 'fail'. Ideally, the verb should be repeated (or a synonym used) tomake all four items parallel – such as 'are unable to deal with...'. The remaining statements – B, C and D – are fully correct. Hence, [2].

Question: 96The passage given below is followed by a set of questions. Choose the most appropriate answer to each question.

Europeans and Americans of ordinary literacy must recognize and write around 52 alphabetic signs (26 capital letters and their lower-caseequivalents), as well as numerals, punctuation marks and a small number of logograms (non-phonetic signs representing words). LiterateJapanese readers, by contrast, with the most complicated writing system in the world, are supposed to know and be able to write twodifferent syllabaries ('kana') with about 50 signs each, plus just under 2,000 further signs ('kanji') taken from Chinese characters, which aregenerally logograms. Those who are highly educated must recognize many more kanji than this. Before the Second World War, therewere some 7,500 kanji in the type-font of Japanese newspapers; even today, newspapers use about 3,200–3,300 kanji.These two situations for readers and writers in Europe/America and in Japan, appear to be poles apart. In fact, the different writingsystems resemble each other more than it seems at first glance. Contrary to what many people think, all scripts that are full writing operateon one basic principle. Both alphabets and the Chinese and Japanese scripts use symbols to represent sounds; and all writing systemsmix such phonetic symbols with logograms. What differs between writing systems –apart from the forms of their signs, of course –are theproportions of the phonetic signs and the logograms.The higher the proportion of phonetic representation in a script, the easier it is to guess the pronunciation of a word from its spelling.Scripts can be classified based on the proportions of phoneticism and logography in them. At one extreme, the International PhoneticAlphabet aims to be purely phonetic. Invented in the late 19th century, its symbols, based on the Roman and Greek alphabets with theaddition of some special symbols and diacritical marks, are intended to be able to write any language with a strict one-to-onecorrespondence between symbol and sound. At the other extreme, cryptographic codes are purely logographic.Among the writing systems used to write spoken languages, the Finnish script has the highest proportion of phoneticism, while theJapanese has the lowest proportion. Hebrew and Arabic, which in their original forms did not mark vowels (though their modern forms do),

3) A, C & E

4) B & E

1) A & D

2) B, C & D

3) A, C & E

4) B & E

Login ID:51A07053/Student Name:ANAND RAJ/Overall Score:0

© CopyrightIMS Learning Resources Pvt.Ltd.,Mumbai.All copyrights to this material vestswith IMS Learning Resources Pvt.Ltd.No part of this materials either in part oras a whole shall be copied,printed,electronically reproduced,sold or distributed without the writtenconsent of IMS Learing Resources Pvt.Ltd.and any such violation would entail initiation of suitable legal proceedings.

Page 76: Login ID:51A07053/Student Name:ANAND RAJ/Overall Score:0 · (i n sq. cm) of such a triangle? Explanation: Question: 9 Answer the question independently. Eight colleagues, four men

lie in the middle. Japanese is adjudged to be slightly more logographic than Chinese, because many Japanese kanji can take multiplevariant readings as a result of their complex history of being borrowed from Chinese to write a different language. On the other hand,Japanese uses the highly phonetic kana syllabary, which has no direct equivalent in Chinese (which nevertheless has syllabic signs). So itis possible to regard Chinese as more logographic than Japanese, depending on which aspects of their scripts are under consideration.From the above discussion, it should be plain that there is no such thing as a 'pure' writing system, that is, a full writing system capable ofexpressing meaning entirely through alphabetic letters or entirely through syllabic signs or entirely through logograms. All systems aremixtures of phonetic and logographic representation. How best to classify writing systems is therefore a controversial matter.In a hypothetical language, the writing system is such that after learning certain basic rules about how the letters correspond to the spokenform, a person learning that language can pronounce the written words correctly most of the time. What can this language be describedas, based on the distinctions given in this passage?Note: Your answer should be a letter. Use the virtual keyboard provided to enter your answer.A. Predominantly phoneticB. Predominantly logographicC. Roughly halfway between phonetic and logographic[quizky-text]

Explanation:Refer to paragraph 3: 'The higher the proportion of phonetic representation in a script, the easier it is to guess the pronunciation of a word

from its spelling.' So in this hypothetical scenario, the language can be considered predominantly phonetic, as the person learning it canfigure out how to pronounce it correctly most of the time after learning certain basic rules. Hence, A.

Question: 97The passage given below is followed by a set of questions. Choose the most appropriate answer to each question.

Which of the following statements is/are true, based on this passage? Choose as many statements as apply.

Note: Your answer should be in alphabetical order. Use the virtual keyboard provided to enter your answer.

A. Modern written Hebrew and Arabic are more phonetic than their original forms.B. Highly educated Japanese people are supposed to know around 7,500 kanji.C.The International Phonetic Alphabet can be used to write any language phonetically.D. All writing systems that are used to write spoken languages are a mixture of phonetic and logographic signs.E. While Japanese has syllabic letters in addition to logographic symbols, Chinese has only the latter.[quizky-text]

Explanation:Refer to paragraph 4: the original forms of Hebrew and Arabic did not mark vowels, while their modern forms do, so A is true. B cannot be

inferred from the passage: according to the first paragraph, highly educated Japanese people are supposed to be able to recognize manymore than the 2,000 kanji that are commonly used, but the 7,500 figure is not related to this (it is the number of kanji used in Japanesenewspapers before the Second World War). C is true, as it is clearly stated in the description of the International Phonetic Alphabet inparagraph 3. D is inferable from paragraph 2. E is incorrect: according to paragraph 4, Chinese also has syllabic signs. Therefore, only A,C and D are true. Hence, ACD.

1) A

2) B

3) C

4) D

1) ACD

2) BCD

3) ADB

4) CDB

Login ID:51A07053/Student Name:ANAND RAJ/Overall Score:0

© CopyrightIMS Learning Resources Pvt.Ltd.,Mumbai.All copyrights to this material vestswith IMS Learning Resources Pvt.Ltd.No part of this materials either in part oras a whole shall be copied,printed,electronically reproduced,sold or distributed without the writtenconsent of IMS Learing Resources Pvt.Ltd.and any such violation would entail initiation of suitable legal proceedings.

Page 77: Login ID:51A07053/Student Name:ANAND RAJ/Overall Score:0 · (i n sq. cm) of such a triangle? Explanation: Question: 9 Answer the question independently. Eight colleagues, four men

Question: 98The passage given below is followed by a set of questions. Choose the most appropriate answer to each question.

This passage is about:

Explanation:This passage is about the scripts used by various languages, not the languages themselves. So [3] can be ruled out at once. While the

passage does discuss the similarities and differences between various scripts, it focuses specifically on one aspect of them: theirproportion of phoneticism and logography. So [4] is too general. There is no reason mentioned in the passage for this difference in scripts,so [2] is incorrect as well. The passage focuses mainly on classifying the writing systems of various languages based on where they fall inthe phonetic to logographic continuum. Hence, [1].

Question: 99Read the short passage given below and answer the question that follows.

We know that blind evolutionary processes can produce human-level general intelligence, since they have already done so at least once.Evolutionary processes with foresight –that is, genetic programs designed and guided by an intelligent human programmer –should beable to achieve a similar outcome with far greater efficiency. This observation has been used by some philosophers and scientists toargue that human-level artificial intelligence (AI) is not only theoretically possible but feasible within this century. The idea is that we canestimate the relative capabilities of evolution and human engineering to produce intelligence, and find that human engineering is alreadyvastly superior to evolution in some areas and is likely to become superior in the remaining areas before too long. The fact that evolutionproduced intelligence therefore indicates that human engineering will soon be able to do the same.Which of the following, if true, most weakens the given argument?

Explanation:The paragraph does not claim that the human engineers intend to use the exact same methods for developing artificial intelligence as

evolution did for human intelligence, so [1] may or may not affect the argument. There is no comparison in the paragraph between theprinciples of human intelligence and artificial intelligence, just the methods of developing the two, so [2] is irrelevant. [4] does not weakenthe argument at all, as the argument is that since, unlike evolution, humans have foresight, it won't take them as long as evolution toproduce intelligence. Only [3] weakens the argument: it suggests that the areas in which human engineering is already superior toevolution were the easiest problems for the engineers to solve, and thus were solved quickly, while the truly difficult ones have remainedintractable the longest, so it is not likely that they will be solved soon. Hence, [3].

Question: 100Read the short passage given below and answer the question that follows.

We know that blind evolutionary processes can produce human-level general intelligence, since they have already done so at least once.Evolutionary processes with foresight –that is, genetic programs designed and guided by an intelligent human programmer –should beable to achieve a similar outcome with far greater efficiency. This observation has been used by some philosophers and scientists toargue that human-level artificial intelligence (AI) is not only theoretically possible but feasible within this century. The idea is that we can

1) The classification of writing systems based on how phonetic or logographic they are.

2) The reason that scripts differ in terms of their proportion of phoneticism and logography.

3) The difference between languages with phonetic scripts and those with logographic ones.

4) The similarities and differences between the writing systems of a wide variety of languages.

1) The precise methods used by evolution to produce human intelligence are not necessarily known to human beings.

2) Human intelligence and artificial intelligence work on very different principles, so comparing the two is not helpful.

3)The areas in which human engineers have thus far failed to match evolution are the ones which they have been working at

the longest.

4) Human engineering may be superior to evolution in some ways, but evolution has had a long time to produce intelligence.

Login ID:51A07053/Student Name:ANAND RAJ/Overall Score:0

© CopyrightIMS Learning Resources Pvt.Ltd.,Mumbai.All copyrights to this material vestswith IMS Learning Resources Pvt.Ltd.No part of this materials either in part oras a whole shall be copied,printed,electronically reproduced,sold or distributed without the writtenconsent of IMS Learing Resources Pvt.Ltd.and any such violation would entail initiation of suitable legal proceedings.

Page 78: Login ID:51A07053/Student Name:ANAND RAJ/Overall Score:0 · (i n sq. cm) of such a triangle? Explanation: Question: 9 Answer the question independently. Eight colleagues, four men

estimate the relative capabilities of evolution and human engineering to produce intelligence, and find that human engineering is alreadyvastly superior to evolution in some areas and is likely to become superior in the remaining areas before too long. The fact that evolutionproduced intelligence therefore indicates that human engineering will soon be able to do the same.Which of the following, if true, most strengthens the given argument?

Explanation:Option [1] is already stated in the paragraph, and as such adds nothing new to the argument. So it cannot be said to strengthen the

argument. [4] is utterly irrelevant to the argument, as human engineers are trying to produce artificial intelligence not recreate humanintelligence, so how well they understand the latter has no bearing on their efforts. The argument is that human engineering will be able toproduce artificial intelligence specifically within this century, not just that it will be able to do so at some point. So [3], which talks about thegoal being achieved 'within a few centuries', in fact weakens the argument slightly rather than strengthening it. The argument in theparagraph is that human engineering can be much more efficient than evolution at developing Intelligence, because evolution is a blindprocess, whereas human engineers have foresight. So [2], which provides examples of technologies that human beings developedseveral magnitudes faster than evolution did, strengthens the argument. Hence, [2].

1) There are areas in which human ingenuity has not only matched but even surpassed evolution.

2)Several technologies, such as heavier-than-air flight and sonar, took humans only centuries or decades to achieve, while

they took evolution billions of years.

3)At the current pace of technological progress, it is possible that machines with human-level artificial intelligence will be

commonplace within a few centuries.

4) Human engineers understand how artificial intelligence works much better than they understand human intelligence.

Login ID:51A07053/Student Name:ANAND RAJ/Overall Score:0

© CopyrightIMS Learning Resources Pvt.Ltd.,Mumbai.All copyrights to this material vestswith IMS Learning Resources Pvt.Ltd.No part of this materials either in part oras a whole shall be copied,printed,electronically reproduced,sold or distributed without the writtenconsent of IMS Learing Resources Pvt.Ltd.and any such violation would entail initiation of suitable legal proceedings.


Recommended